Sei sulla pagina 1di 80

Volume 24 No.

8 August 2016
Managing Editor Corporate Office:
Mahabir Singh Plot 99, Sector 44 Institutional area, Gurgaon -122 003 (HR).
Editor Tel : 0124-4951200 e-mail : info@mtg.in website : www.mtg.in
Anil Ahlawat Regd. Office:
(BE, MBA) 406, Taj Apartment, Near Safdarjung Hospital, New Delhi - 110029.

AIIMS Topper Interview 8


Class 11

CONTENTS
NEET | JEE Essentials 12
Ace Your Way CBSE 24
MPP-2 34
Brain Map 46
Class 12
NEET | JEE Essentials 38
Brain Map 47
Ace Your Way CBSE 52
Core Concept 61
Exam Prep 67
MPP-2 76
Competition Edge
Physics Musing Problem Set 37 80
You Ask We Answer 82
Live Physics 83
Physics Musing Solution Set 36 84
Crossword 85
subscribe online at www.mtg.in
individual subscription rates combined subscription rates
1 yr. 2 yrs. 3 yrs. 1 yr. 2 yrs. 3 yrs.
Mathematics Today 330 600 775 PCM 900 1500 1900
Chemistry Today 330 600 775 PCB 900 1500 1900
Physics For You 330 600 775 PCMB 1000 1800 2300
Biology Today 330 600 775
Send D.D/M.O in favour of MTG Learning Media (P) Ltd.
Payments should be made directly to : MTG Learning Media (P) Ltd,
Plot No. 99, Sector 44, Gurgaon - 122003 (Haryana)
We have not appointed any subscription agent.
Owned, Printed and Published by Mahabir Singh from 406, Taj Apartment, New Delhi - 29 and printed
by Personal Graphics and Advertisers (P) Ltd., Okhla Industrial Area, Phase-II, New Delhi. Readers are
adviced to make appropriate thorough enquiries before acting upon any advertisements published in this
magazine. Focus/Infocus features are marketing incentives. MTG does not vouch or subscribe to the claims
and representations made by advertisers. All disputes are subject to Delhi jurisdiction only.
Editor : Anil Ahlawat
Copyright MTG Learning Media (P) Ltd.
All rights reserved. Reproduction in any form is prohibited.

Physics For you | AUGUST 16 7


Nikhil Bajiya, AIR 2 at AIIMS MBBS 2016 exam dreamt of being a doctor from

Topper Interview
his very childhood and with determination and perseverance, he achieved the
top rank in the highly competitive medical entrance exam. With dedicated
preparation for 2 years, he hardly let anything distract him from his aim.
The aspiring neurosurgeon is aiming to take admission in AIIMS Delhi. Hailing
from Jhunjhunu, Rajasthan, Nikhil went to G.B Modi Vidya Mandir and moved
to Kota, the hub of coaching institutes for competitive entrance exams. He
believes that self-study is equally important along with the guidance of
coaching mentors and followed a systematic preparation plan. Nikhil would
be the second doctor in his family after his sister who is presently pursuing
medicine. He loves to unwind by playing badminton, table tennis or computer
AIIMS 2016
games.

Many congratulations for your Nikhil Bajiya: No I was not much


achievement in cracking the bothered about the AIPMT and NEET
AIIMS MBBS entrance exam! exam row. I had aimed for AIPMT/
Were you expecting to be among NEET and AIIMS and I was very
the top rankers? confident about cracking the exams
Nikhil Bajiya: Many thanks for your and get admission to a good college
wishes! I was confident about my since my preparation was moving in
performance and had expected to right direction. At the end of the day,
be among the top rankers but never all that matters is how you perform,
expected to be the AIR 2. Also, to results reflect automatically. I ensured
be frank, I never speculated about that I do not take any undue stress
the rank. I had only focused on my during the preparation phase.
performance and after the exam, I did Tell us something about your
not think about the results. family members and their
Why did you decide to pursue contributions in your success.
medicine? Nikhil Bajiya: My family consists
Nikhil Bajiya: I was inspired to of my parents and elder sister. Their
be a doctor from my paternal uncle contributions behind my success are
since childhood. I was motivated by enormous and I cant express them
his practice as a pediatrician and the in words. Since I had joined coaching
respect he earned. I made sure that I in Kota, my father, who is the Deputy
work hard to reach my goal. Registrar with the Cooperative
Which other medicine entrance Societies Department took transfer in
exam did you appear for? the city just to be with me. My mother
Nikhil Bajiya: Apart from AIIMS is a government school teacher and
MBBS 2016, I had appeared for the my elder sister is studying medicine
NEET 1 exam. in Bikaner. All of them have been
immensely supportive throughout.
This year, there has been lot of
confusion regarding AIPMT and What was your preparation
NEET. Did the ongoing NEET row strategy for the exam?
Nikhil Bajiya
affect your preparation? Nikhil Bajiya: I had started full-
fledged preparation from Class XI. I had joined a Kota based would like to follow?
coaching institute for medical entrance exam preparation. I had Nikhil Bajiya: I plan to take admission in AIIMS Delhi. I want
strictly followed the guidelines given by my faculty members. to be a neurosurgeon.
Initially I concentrated on gaining stronghold
Systematic preparation You had followed a strict study schedule.
over the concepts followed by taking regular How did you unwind yourself? Could
strategy and focus
practice tests and solving previous years you find time to pursue your hobbies?
are key factors for my
question papers and taking lessons of Nikhil Bajiya: Since I stayed in Kota, away
success, says Nikhil
improvement from the mistakes. Apart from Bajiya, AIR 2 from family, I did not find my scope to pursue
coaching, I used to put around 6 to 7 hours my hobby, which is a sport. I love playing
of self-study. badminton, table tennis and computer games. Occasionally I
What were your strong and weak areas among Physics, played these games whenever I found some time. Otherwise
Chemistry and Biology? I used to take short naps within the day to catch up on my
Nikhil Bajiya: Physics have always been my strong point and sleep.
it is my favourite subject. However, that does not mean I am What is your message for the medicine aspirants who
weak in any of the two other subjects. I am equally good in would be appearing for the exam next year?
Chemistry and Biology too and there is no such weak subject Nikhil Bajiya: I would suggest them to never over burden
or areas for me. themselves. Follow a systematic preparation schedule with
Which is your dream medical college where you aspire ample practice. Courtesy : careers360.com

to take admission? Is there any specialization you

10 Physics For you | AUGUST 16


NEET JEE
Class
XI

ESSENTIALS


Unit
2 Kinematics
) Motion If all three coordinates change with time, motion
Motion is a combined property of the object under is three dimensional (3 - D) or motion in space.
study and the observer. There is no meaning of rest ) Distance anD DisPlaceMent
or motion without the viewer.
Total length of path C
If a body changes its position with time, it is said to covered by the particle,
be moving else it is at rest. Motion is always relative
in definite time interval
to the observer.
is called distance. The A B
To locate the position of a particle we need a length of path ACB is called the distance travelled
reference frame. A commonly used reference frame by the body.
is cartesian coordinate system or x-y-z coordinate
Overall, body is displaced from A to B. A vector
system.
for A to B i.e., AB is its displacement vector
The coordinates (x, y, z) of the particle specify
or displacement that is the minimum distance.
the position of the particle with respect to
Displacement is directed from initial position to
origin of that frame.
final position.
If all the three coordinates of the particle remain
unchanged as time passes it means the particle For a moving body, distance can not have zero or
is at rest with respect to this frame. negative values but displacement may be +ve, ve
or zero.
The reference frame is chosen according to
problems. If motion is in straight line without change in
direction, then
If frame is not mention, then ground is taken as
reference frame. distance = |displacement|
If only one coordinate changes with time, motion If motion is not in straight line, then
is one dimensional motion (1- D) or rectilinear distance > |displacement|
motion. Magnitude of displacement may be equal or less
If only two coordinates change with time, motion than distance but never greater than distance.
is two dimensional (2 - D) or motion in a plane. distance |displacement|

12 Physics For you | august 16


Displacement in terms of position vector : average velocity
Let a body is Y Velocity is the rate of change of position vector
displaced from A(x1, y1, z1) or change in position per unit time.

A(x1, y1, z1) to r Suppose a particle Y

B(x2, y2, z2) then
rA rB B(x2, y2, z2) is at a point A at A
its displacement is time t1 and B at
X
r2
given by vector AB. O time t2. Position r1 B
From DOAB vectors of A and B O X
Z
are r1 and r2 . The
rA + AB = rB or AB = rB rA displacement in this Z

Q rB = x2 i + y2 j + z2 k and rA = x1 i + y1 j + z1 k time interval is the vector AB = (r2 r1 ). The
average velocity in this time interval is,
\ AB = (x2 x1 )i + ( y2 y1 )j + (z2 z1 )k
displacement
or AB = Dxi + Dy j + Dzk vav =
time interval

) sPeeD anD velocity AB r r
Speed is related to distance and it is a scalar while vav = = 2 1
t t t t
velocity is related to displacement and it is a vector. 2 1 2 1
For a moving body speed can not have zero or Here, AB = r2 r1 = change in position vector.
For small time interval between t and t + Dt,
negative values but velocity can have.
change in position vector is Dr then average
average speed
Dr
It is defined for a time interval. Average speed velocity in Dt time interval is, vav =
of a trip Dt
instantaneous velocity
Total distance travelled It is the velocity at a particular instant. If
vav =
Total time taken time interval approaches zero then average
Let Ds be the distance travelled in the time velocity become instantaneous velocity.

interval t to t + Dt. The average speed in this Dr dr
v = lim =
time interval is Dt 0 Dt dt
Ds i.e., instantaneous velocity is the time derivative
vav = of position vector.
Dt
If a particle travels distances s1, s2, s3 etc. with Magnitude of instantaneous velocity is the

speeds v1, v2, v3 etc. respectively, then total instantaneous speed.


distance travelled s = s1 + s2 + s3 + ...... + sn When a particle moves with constant velocity,

s s s s its average velocity, its instantaneous velocity


Total time taken = 1 + 2 + 3 + .... + n and its speed all are equal.
v1 v2 v3 vn
s + s + s + ..... + sn ) acceleration
Average speed of a trip = 1 2 3 The acceleration is rate of change of velocity or
s1 s2 sn
v + v + ..... + v change in velocity per unit time interval.
1 2 n Velocity is a vector quantity hence a change in its
instantaneous speed
magnitude or in direction or in both, will give the
The speed at a particular instant is defined as
acceleration (or non uniform motion).
instantaneous speed (or speed).
If Dt approaches zero, average speed becomes
average acceleration

instantaneous speed. Let velocity of a particle at t1 is v1 and at

Ds ds t2 it is v2 . The change in velocity in time
v = lim =
interval (t2 t1) is (v2 v1 ).
Dt 0 Dt dt
i.e., instantaneous speed is the time derivative v2 v1
\ aav = .
of distance. t2 t1

Physics For you | august 16 13


For small time interval between t and t + Dt, 1
s= (u + v)t ..... without a
change in velocity is taken Dv then average 2
th a
Dv Distance travelled in n sec, sn = u + (2n 1)
acceleration in the time interval Dt is, aav = . 2
Dt If the motion is not along a straight line
instantaneous acceleration
v = u + at
If Dt approaches zero, the average acceleration
1
becomes the instantaneous acceleration (or s = ut + at 2
2
Dv dv
acceleration) a = lim = v v = u u + 2a s
Dt 0 Dt dt

i.e., instantaneous acceleration is the time Equations of motion are valid only when
derivative of velocity. acceleration remains constant during motion,
otherwise we have to write the equations as under :
) equations oF Motion v = a(t ) dt and s = v(t ) dt

If u = initial velocity of the body,
) eFFective use oF MatheMatical
a = uniform acceleration of the body,
tools in solvinG ProBleMs oF one-
s = displacement in time t.
DiMensional Motion
v = velocity of the body after a time t, then
the following equations hold good, in order to If displacement-time equation is given, we can
describe the motion of the body. get velocity-time equation with the help of
differentiation. Again, we can get acceleration-time
If the motion is along a straight line,
equation with the help of differentiation.
v = u + at ..... without s
If acceleration-time equation is given, we can
1 2 get velocity-time equation by integration. From
s = ut + at ..... without v
2 velocity-time equation, we can get displacement-
2 2
v = u + 2as ..... without t time equation by integration.
1 2
s = vt at ..... without u
2
) KineMatic GraPhs
Displacement-time Graph for various types of Motion of a Body
Description of motion Shape of graph Feature of graph
Displacement

For a stationary body, the displacement-time The slope of straight line AB


graph is a straight line AB parallel to the time A
s = constant
B
(representing instantaneous velocity)
axis. is zero.
O Time

A
Displacement

When a body is moving with constant velocity,


Greater is the slope of straight line OA,
the displacement-time graph will be a straight s = vt greater will be the velocity.
line OA, inclined to time axis.
O Time
Displacement

When a body is moving with a constant 1 2 The slope of displacement-time curve


s= at
acceleration, the displacement-time graph is a 2 (i.e., instantaneous velocity) increases
curve which bend upwards. with time.
O Time

14 Physics For you | august 16


Displacement
When a body is moving with constant 1 2 The slope of displacement-time curve
s = at
retardation, the displacement-time graph is a 2 (i.e., instantaneous velocity) decreases
curve which bend downwards. with time.
O Time
B
s=

Displacement
When a body is moving with infinite velocity,
Such a motion of the body is never
the displacement-time curve is a straight line
possible.
AB parallel to displacement axis.
O A Time
When a body returns back towards the original

Displacement
point of reference while moving with uniform A
s = vt The displacement of the body decreases
negative velocity, the displacement-time graph with time with respect to the reference
is an oblique straight line AB, making an angle > 90 point, till it becomes zero.
q > 90 with the time axis. O B Time

velocity-time Graph for various types of Motion of a Body


Description of motion Shape of graph Feature of graph

When a body is moving with a constant velocity, The slope of this graph (representing
v = constant
Velocity

the velocity-time graph is a straight line AB A B the instantaneous acceleration) is


parallel to time axis. zero.
O Time

A
When a body is moving with a constant
Greater is the slope of straight line
acceleration and its initial velocity is zero, the
Velocity

v = at OA, greater will be the instantaneous


velocity-time graph is an oblique straight line,
acceleration.
passing through the origin.
O Time

at B (i) Here OA represents the initial


When a body is moving with a constant +
=u velocity of the body.
acceleration and its initial velocity is not zero, v
Velocity

A (ii) The area enclosed by the velocity-


the velocity-time graph is an oblique straight
time graph with time axis represents
line AB not passing through the origin.
O Time the distance travelled by the body.

When a body is moving with increasing The slope of velocity-time graph


v = kt 2
Velocity

acceleration, the velocity-time graph is a curve (i.e. instantaneous acceleration


which bend upwards. increases with time.
O Time

When a body is moving with a constant A


retardation and its initial velocity is not zero, v = at The slope of this straight line with
Velocity

the velocity-time graph is an oblique straight B time axis, makes an angle q > 90.
line not passing through the origin.
O Time

Physics For you | august 16 15


acceleration-time Graph for various types of Motion of a Body

Description of motion Shape of graph Feature of graph

Acceleration
When a body is moving with constant The area enclosed by acceleration-
acceleration, the acceleration-time graph is a A
a = constant
B
time graph for the given time gives
straight line AB parallel to time axis. the velocity of the body.
O Time

A
The body is moving with constant

Acceleration
When a body is moving with constant
kt acceleration and slope of straight
increasing acceleration, the acceleration-time a=
line OA, makes an angle q < 90 with
graph is a straight line OA.
time axis.
O Time

The body is moving with negative


Acceleration

When a body is moving with constant


a = kt acceleration and slope of straight
decreasing acceleration, the acceleration-time
line makes an angle q > 90 with time
graph is a straight line.
axis.
O Time

) vertical Motion unDer Gravity Body projected vertically upward


Take initial position as y v=0
If air resistance is neglected and a body is freely origin and direction of
moving along vertical line near the earths surface motion (i.e., vertically (v, t)
h u 0, t = 0
then an acceleration acts downward which is upward) as positive y-axis. y = 0
9.8 m s2 or 980 cm s2. v = 0 at maximum height, at t = T,
a = g (because directed downward)
Freely falling body released from a height h
Equations of motion of the particle at any latter
above the ground time t become
Taking initial position as u = 0, t = 0 v = u gt
origin and direction of h 1
h = ut gt2
motion (i.e., downward v 2
y
direction) positive y-axis. v2 = u2 2gh
As body is just released/dropped, u = 0 At time t = T, u = gT
acceleration along +y axis, a = g 1 2hmax
hmax = gT 2 or, T =
The body acquires velocity v(downward) after 2 g
falling a distance h in time t, then equations of u2
u2 = 2ghmax or, hmax =
motion become 2g
After attaining maximum height, body comes back
v
v = gt or t = g at the ground. During complete flight acceleration
is constant, a = g.
Time taken during up flight
1 2 2h y
v=0
h= gt or t = g and down flight are equal.
2 u
Time for one side, T =
g
v2 = 2gh (or v = 2 gh or h = v 2 / 2 g ) and total flight time = 2T =
2u u
h

16 Physics For you | august 16



At each equal height from ground speed of body If the man swims across the river i.e., v and v R
will be same whether going up or coming down. are not collinear then use the vector algebra
) relative Motion vR


There is no meaning of motion without reference or vm = v + v R
v

observer. If reference is not mentioned then we take vm


the ground as a reference of motion. Velocity or


displacement of the particle with respect to ground Case (I) : For shortest path, resultant velocity

is called actual velocity or actual displacement of vm = (v + v R ) is in the direction of displacement
the body. AB. B
To reach at B, v sin q = vR
If we describe the motion of a particle with respect vR

v d
sin q = R vm

to an object which is also moving w.r.t. ground then


v v v cos = vm
velocity of particle w.r.t. ground is its actual velocity

Component of velocity v sin = v A


(vact ) and velocity of particle w.r.t. moving object is along AB = v cos q
R
(For mimimum displacement)
its relative velocity (vrel ) and the velocity of moving d d
So time taken, T = =
object (w.r.t. ground) is the reference velocity (vref ). v cos q v2 v2 R
In the figure let S is ground frame and S is frame
Case (II) : For minimum B C
of moving object. Position of particle P relative to
time, man should start
frame S is rPS while position of frame S relative to swimming perpendicular vR

d
frame S is rS S at a moment. According to vector to water current. Due to
v v

law of addition rPS = rPS + rS S . effect of river velocity, m


A
Differentiate the equation w.r.t. time, man will reach at point C (For minimum time)
S S along resultant velocity, i.e., his displacement will
drPS drPS drS S dr P
= + but v = not be minimum but time taken to cross the river
dt dt dt dt


rPS will be minimum.
rPS
So, v PS = v PS + vS S d
t min =
i.e., vactual = v relative + vreference rSS v
In time tmin swimmer travels distance BC along the
or vrelative = vactual vreference river with speed of river vR.
swimming in the river \ BC = tmin vR = distance travelled along river flow
d
A man can swim with velocity v i.e., the = drift of man = v R .
v
velocity of man w.r.t. still water. If water is also
) Projectile Motion
flowing with velocity v R then velocity of man
A particle thrown in the space which moves under
relative to ground vm = v + v R
the effect of gravity only is called a projectile. The
If the swimming is in the direction of flow of
motion of this projectile is referred as projectile
water or along the downstream, then motion.
v
Projectile motion = Horizontal motion + Vertical
vm = v + vR motion
vR

Angle between velocity vector and acceleration


If the swimming is in the direction opposite to vector during the flight.
the flow of water or along the upstream, then v1
v2
v3
u
v

g v4
vm = v vR vR

g
g
g
g

Physics For you | august 16 17


In ascending motion q is an obtuse angle. At the 2uy u2y 2ux u y
top of the flight q = 90, and during descending T= ,H = ,R=
g 2g g
motion, q is an acute angle. During the flight
T and H depend only upon initial vertical
q decreases continuously with increase in time
speed uy.
and it always lies between 0 and 180.
If two projectiles thrown in different directions,
In projectile motion, magnitude as well as direction
have equal time of flight then their initial
of velocity continuously changes so there must
vertical speeds are same so that their maximum
be presence of tangential (at) as well as radial
height attained is also same.
component (ar) acceleration.
If HA = HB then (uy)A = (uy)B and TA = TB
at = 0
v vx Y

ar = g
ar va = g sin H HA HB
= t
at = g sin gc
g os g
ar = g cos X
O
In ascending motion, at is against v, so v decreases When two projectiles are thrown with equal speeds
and in descending motion at is in the direction of at angle q and (90 q), then their ranges are equal
v, so v increases. but maximum heights attained are different and
Since deceleration in y-component of velocity time of flights are also different.
in ascending motion is equal to acceleration in For maximum range, q = 45
y-component of velocity during descending motion, u2 sin 2(45) u2 sin 90 u2
so time of ascending motion is equal to the time of and Rmax = = Rmax =
g g g
descending motion and magnitude of y-component
of velocity at same height in ascending as well as
in descending motion is same but opposite in
directions.
At maximum height, vy = 0 and vx = ux = ucosq so
that at maximum height v = v 2x + v 2y = u cos q
When particle again returns to ground at point u2 sin2 45 u2
Here H = =
B, its y coordinate is zero and its magnitude of 2g 4g
velocity is u at angle q with ground. Angular change Rmax
\ = 4 Rmax : H = 4 : 1
(or difference) between initial velocity and final H
velocity = 2q. For R = H
Y u2 (2 sin q cos q) u2 sin2 q sin q
A v = u cos = 2 cos q =
u g 2g 2
H tanq = 4 \ q = tan1(4) = 76
B
O X ) circular Motion

u When a particle moves in a plane such that its
distance, from fixed (or moving) point remains
Initial velocity ui = u cos qi + u sin qj
constant then its motion is called as circular motion
Final velocity, u f = u cos qi u sin qj with respect to that fixed (or moving) point. That

Total change in its velocity =| Du | = 2u sin q point is called centre and the distance is called

Total change in momentum = m | Du | = 2mu sin q radius of circular path.
If K0 is initial kinetic energy, then kinetic energy at Angular Displacement : Angle traced by position
highest point = K0cos2q vector of a particle moving w.r.t. some fixed point is

18 Physics For you | august 16


called angular displacement. Angular Acceleration
Q
Dq = angular displacement Fixed Rate of change of angular velocity is called angular
point acceleration.
Arc ArcPQ
Angle = ; Dq = r
Radius r P Dw dw dw
= lim = ; =
Dt 0 Dt dt dt
Small angular displacement dq is a vector
quantity, but large angular displacement is It is an axial vector quantity. Its direction is along
scalar quantity. the axis according to right hand screw rule.
Its direction is perpendicular to plane of Relation between Angular and Linear
rotation and given by right hand screw rule. Acceleration

It is dimensionless and its S.I. unit is radian v = w r ( v is a tangential vector, w is a axial vector

while other units are degree or revolution. and r is a radial vector.)
2p radian = 360 = 1 revolution
Angular Velocity : It is defined as the rate of change
of angular displacement of moving particle.
Angle traced Dq dq
w= = lim =
Time taken Dt Dt dt

Its direction is same as that of angular dv d dw dr
but a = = (w r ) = r +w
displacement i.e., perpendicular to the plane of dt dt dt dt

rotation and along the axis according to right or a = r + w v

hand screw rule. or a = aT + aC

Its unit is radian/second. ( aT = r is tangential acceleration and

Relation between Linear and Angular Velocity aC = w v is centripetal acceleration)

Arc Q aT and aC are two mutually perpendicular
Angle = r
Radius s components of net linear acceleration.
Ds r
Dq = or Ds = r Dq ) uniForM circular Motion
r P
Ds r Dq ds dq When a body moves along a circular path with
\ = if Dt 0 then = r or v = wr uniform speed, its motion is said to be uniform
Dt Dt dt dt
In vector form circular motion.

v = wr Position vector (r ) is always perpendicular to

(direction of v is found according to right hand the velocity vector (v ) i.e. r v = 0
rule) Velocity vector is always perpendicular to the

Average Angular Velocity (wav) acceleration. v a = 0
Total angle of rotation q2 q1 Dq For circular motion, force
wav = = = towards centre (centripetal
Total time taken t2 t1 Dt
force) must act so that
2p
= = 2pn direction of v keeps on
T changing.
where q1 and q2 are angular positions of the particle
The work done by centripetal force is always zero.
at instants t1 and t2 respectively.
Kinetic energy = constant
Instantaneous Angular Velocity
Since | v | = constant, so tangential acceleration at = 0
The angular velocity at some particular instant In projectile motion, both the magnitude and the
of time is called instantaneous angular velocity. direction of acceleration (g) remain constant, while

Dq dq dq in uniform circular motion the magnitude remains
w = lim = ; w=
Dt 0 Dt dt dt constant but the direction continuously changes.

Physics For you | august 16 19


1. A ball falls from 20 m height on the floor and 10 minutes. What are the values of x and v ?
rebounds to 5 m. Time of contact is 0.02 s. Find the (a) x = 15 min, v = 90 km h1
acceleration during impact. (b) x = 13 min 20 s, v = 90 km h1
(a) 1200 m s2 (b) 1000 m s2 (c) x = 15 min, v = 75 km h1
2
(c) 2000 m s (d) 1500 m s2 (d) x = 13 min 20 s, v = 70 km h1
2. The displacement-time (x-t) graph 7. A particle is projected with a certain velocity at two
of a body is shown in figure. The different angles of projection with respect to the
body is accelerated along the horizontal plane so as to have the same range R on
path the horizontal plane. If t1 and t2 are the times taken
(a) OA only for the two paths, then which one of the following
(b) BC only relations is correct?
(c) CD only (a) t1t2 = 2R/g (b) t1t2 = R/g
(d) OA and CD only (c) t1t2 = R/2g (d) t1t2 = 4R/g
v(m s1) 8. The position of a particle moving in the x-y plane
3. The velocity versus
time graph of a body at any time t is given by; x = (3t2 6t) metres;
moving in a straight y = (t22t) metres. Select the correct statement.
line is shown in figure. (a) Acceleration is zero at t = 0
The displacement of the (b) Velocity is zero at t = 0
body in 5 s is (c) Velocity is zero at t = 1 s
(d) Velocity and acceleration of the particle are
(a) 3 m (b) 5 m (c) 4 m (d) 2 m
never zero.
4. An aeroplane is flying in a horizontal direction
9. A stone tied to the end of a 1 m long string, is whirled
with a velocity of 360 km h1 and at a height of
in a horizontal circle with a constant speed. If the stone
1960 m. When it is vertically above the point A on
makes 22 revolutions in 44 s, what is the magnitude
the ground, a body is dropped from it. The body
and direction of acceleration of the stone?
strikes the ground at a point B. The distance AB is
p2
(a) 2000 2 m (b) 2000 m (a) m s 2 and direction along the radius towards
4
(c) 1000 2 m (d) 1000 m the centre.
5. A body initially at rest is moving with uniform (b) p2 m s2 and direction along the radius away
acceleration a. Its velocity after n seconds is v. The from the centre.
displacement of the body in last 2 s is (c) p2 m s2 and direction along the radius towards
2v(n 1) v(n 1) the centre.
(a) (b)
n n (d) p2 m s2 and direction along the tangent to the
v(n + 1) 2v(n + 1)
(c) (d) circle.
n n
6. Two cities C1 and C2 are connected on the opposite 10. Water drops fall at regular intervals from a tap
ends of a long straight parallel track. The cities are which is 5.0 m above the ground. The third drop is
connected by a train service as well as a bus service. leaving the tap at the instant the first drop reaches
The trains leave with constant speed v for either city the ground. How far above the ground is the second
at regular frequency of one train every x minute. drop at that instant?
The buses ply on a parallel road at a constant speed of (a) 1.25 m (b) 2.50 m (c) 3.75 m (d) 5.00 m
30 km h1. A bus passenger going from city C1 to city 11. Two cars having masses m1 and m2 move in circles
C2 observes a train going past him every 20 minutes of radii r1 and r2 respectively. If they complete the
while a train goes in the opposite direction every

20 Physics For you | August 16


circle in equal time, the ratio of their angular speed 16. If a body is moving in a circular path maintains
w1/w2 is constant speed of 10 m s1, then which of the
m r m1r1 following correctly describes relation between
(a) 1 (b) 1 (c) m r (d) 1 acceleration and radius ?
m2 r2 22
12. A man running on a horizontal road at 8 km h1
finds the rain falling vertically. He increases his speed
to 12 km h1 and finds that the drops make angle (a) (b)
30 with the vertical. What is the speed of the rain ?
(a) 8 km h1 (b) 4 7 km h 1
(c) 8 3 km h1 (d) 7 km h1
13. A particle moves so that its position vector is given
^ ^
by r = cos wt x + sin wt y , where w is a constant. (c) (d) a
Which of the following is true?

(a) Velocity is perpendicular to r and acceleration
is directed towards the origin. [JEE Main Online 2015]

(b) Velocity is perpendicular to r and acceleration
17. The position vector of a particle R as a function of
is directed away from the origin.
(c) Velocity and acceleration both are time is given by R = 4 sin(2pt )i + 4 cos(2pt )j Where

perpendicular to r . R is in meters, t is in seconds and i and j denote

(d) Velocity and acceleration both are parallel to r . unit vectors along x and y-directions, respectively.
[NEET Phase - 1 2016] Which one of the following statements is wrong for
14. A uniform circular disc of radius 50 cm at rest is free the motion of the particle?
to turn about an axis which is perpendicular to its (a) Magnitude of the velocity of particle is 8 m s1.
plane and passes through its centre. It is subjected (b) Path of the particle is a circle of radius 4 m.
to a torque which produces a constant angular (c) Acceleration vector is along R.
acceleration of 2.0 rad s2. Its net acceleration in v2
(d) Magnitude of acceleration vector is , where
m s2 at the end of 2.0 s is approximately v is the velocity of particle. R
(a) 6.0 (b) 3.0 (c) 8.0 (d) 7.0 [AIPMT 2015]
[NEET Phase - 1 2016]
18. From a tower of height H, a particle is thrown
15. Two stones are thrown up simultaneously from
vertically upwards with a speed u. The time taken
the edge of a cliff 240 m high with initial speed of
by the particle, to hit the ground, is n times that
10 m s1 and 40 m s1 respectively. Which of the
taken by it to reach the highest point of its path.
following graph best represents the time variation
The relation between H, u and n is
of relative position of the second stone with respect
(a) gH = (n 2)u2 (b) 2gH = n2u2
to the first stone?
(c) gH = (n 2)2u2 (d) 2gH = nu2(n 2)
(Assume stones do not rebound after hitting the ground
[JEE Main 2014]
and neglect air resistance, take g = 10 m s2)
19. A projectile is fired from the surface of the earth
with a velocity of 5 m s1 and angle q with the
horizontal. Another projectile fired from another
(a) (b)
planet with a velocity of 3 m s1 at the same angle
follows a trajectory which is identical with the
trajectory of the projectile fired from the earth. The
value of the acceleration due to gravity on the planet
(in m s2) is (Given g = 9.8 m s2)
(c) (d) (a) 3.5 (b) 5.9 (c) 16.3 (d) 110.8
[AIPMT 2014]
_
[JEE Main 2015] 20. A projectile is given an initial velocity of (i + 2 j)m s 1 ,
where, i is along the ground and j is along the vertical.
^

Physics For you | August 16 21


If g = 10 m s2, the equation of its trajectory is i.e., R = v02 sin 2q / g
(a) 4y = 2x 25x2 (b) y = x 5x2 2v sin q 2v sin(90 q) 2v0 cos q
(c) y = 2x 5x2 (d) 4y = 2x 5x2 As t1 = 0 and t2 = 0 = ,
g g g
[JEE Main 2013]
4v 2 sin q cos q 2 v02 sin 2q 2R
SolutionS t1t2 = 0 = =
g2 g g g
1. (d) : The acceleration during impact,
dx
v (v1 ) v2 + v1 2 gh2 + 2 gh1 8. (c) : Since, x = 3t2 6t \ vx = = 6t 6
dt
a= 2 = = At t = 1 s, vx = 6 1 6 = 0
t t t
2 10 20 + 2 10 5 20 + 10 dy
= = = 1500 m s 2 Also, y = t2 2t \ vy = = 2t 2
0.02 0.02 dt
2. (a) : The slope of x-t graph gives velocity and the At t = 1 s, vy = 2 1 2 = 0
slope of the graph is increasing in part OA only. 2
9. (c) : a = r w = r (2pn / t )
2
3. (c) : Displacement = area of bigger triangle area
of smaller triangle + area of rectangle 4 p2n2 1 4 p2 (22)2
=r = = p2 m s 2
2 2
1 1 t (44)
= (3 2) (1 2) + (1 2) = 4 m
2 2 10. (c) : Let T be the time interval between the drops
4. (b) : Time taken by the body to strike the ground is (1, 2, 3) falling from the tap as shown in the figure.
1 2h 2 1960
given by h = gt 2 or t = = = 20 s
2 g 9. 8
Velocity of the aeroplane in horizontal direction,
vx0 = 360 km h1 = 100 m s1
Distance AB = vx0 t = (100 20) m = 2000 m
v u v 0 v
5. (a) : Since, a = = =
t n n Since distance covered by the first drop in time 2T
Displacement in last 2 s, is 5 m,
1 1 1
sn sn2 = an2 a(n 2)2 5 = g (2T )2 = 2 gT 2 ...(i)
2 2 2
v 2v(n 1) Further, distance covered by the second drop in
= 2a(n 1) = 2 (n 1) =
n n time T (from t = T to t = 2T),
6. (b) : If v (in km h1) is the constant speed of the 1
y = gT 2 ...(ii)
trains then the distance between the successive trains 2
x vx From eqns. (i) and (ii), y = 1.25 m
=v = km Distance of the second drop from the ground
60 60
When a train moves in the same direction as that of = 5 y = 5 1.25 = 3.75 m
vx / 60 20 11. (d)
the bus passenger, =
(v 30) 60
12. ( b): We have, vrain, road = vrain, man + vman, road ...(i)
or vx = 20(v 30) ...(i)
When a train moves in a direction opposite to the The two situations given in the problem may be
bus passenger, represented by the given figures.
vx / 60 10
= or vx = 10(v + 30) ...(ii)
(v + 30) 60
From eqns. (i) and (ii),
20(v 30) = 10(v + 30) or v = 90 km h1
From eqn. (i),
90x = 20(90 30) = 1200 or x = 13 min 20 s
7. (a) : R is same for angles of projection q and (90 q),

22 Physics For you | August 16


vrain, road is same in magnitude and direction in stone reaches to the ground.
both the figures. 16. (c) : Speed v = 10 m s1
Taking horizontal components in equation (i) for
We know, centripetal acceleration is given by,
figure (a)
v2
vrain, road sin a = 8 ...(ii) a=
r
Now consider figure (b). Draw a line OA ^ vrain, man
as shown. Q | v | = constant
Taking components in equation (i) along the line OA. 1
vrain, road sin(30 + a) = 12 cos30 ...(iii) so, a or ar = constant
r
From (ii) and (iii), This represents a rectangular hyperbola.
sin(30 + a) 12 3 3 17. (a)
= or cot a =
sin a 82 2 18. (d) : Time taken by the particle to
8 reach the top most point is,
From (ii), vrain, road = = 4 7 km h 1 u
sin a t= ... (i)
g
13. (a) : Given, r = cos wt x + sin wt y Time taken by the particle to reach

dr
\ v= = w sin wt x + w cos wt y the ground = nt
dt
Using, s = ut + 1 at 2
dv 2
a= = w2 cos wt x w2 sin wt y = w2r 1
dt H = u(nt ) g (nt )2
2
Since position vector (r ) is directed away from the
2
origin, so, acceleration (w2r ) is directed towards u 1 u
H = u n gn2 [using (i)]
the origin. Also, g 2 g

r v = w sin wt cos wt + w sin wt cos wt = 0 2gH = 2nu2 n2u2 2gH = nu2(n 2)

r ^v
19. (a) : The equation of trajectory is
14. (c) : Given, r = 50 cm = 0.5 m, a = 2.0 rad s2, w0 = 0
At the end of 2 s, gx 2
y = x tan q
Tangential acceleration, at = ra = 0.5 2 = 1 m s2 2u2 cos2 q
Radial acceleration, ar = w2r = (w0 + at)2r where q is the angle of projection and u is the
= (0 + 2 2)2 0.5 = 8 m s2 velocity with which projectile is projected.
\ Net acceleration, For equal trajectories and for same angles of
a= at2 + ar2 = 12 + 82 = 65 8 m s 2 g
projection, = constant
1 u2
15. (a) : Using h = ut + at2 9. 8 g
2 As per question, =
1 52 32
For first stone, y1 = 10t gt2 where g is acceleration due to gravity on the
2
1 planet.
For second stone, y2 = 40t gt2 9.8 9
2 g = = 3.5 m s 2
Relative position of the second stone with respect to 25
1 1
the first stone Dy = y2 y1 = 40t gt2 10t + gt2 20. (c) : Given : u = i + 2 j
2 2
= 30t As u = ux i + u y j \ ux = 1 and uy = 2
Therefore, it will be a straight line. Also x = uxt
After 8 seconds, first stone reaches to the ground, 1 2
\ x = t and y = u y t gt
i.e., y1 = 240 m 2
1
\ Dy = y2 y1 = 40t gt2 + 240 .. . y = 2t 1 10 t 2 = 2t 5t 2
2 2
Therefore, it will be a parabolic curve till the second Equation of trajectory is y = 2x 5x2.

Physics For you | August 16 23


CLASS XI Series 2

CBSE
Motion in a Plane Time Allowed : 3 hours
Maximum Marks : 70
Laws of Motion
GENERAL INSTRUCTIONS
(i) All questions are compulsory.
(ii) Q. no. 1 to 5 are very short answer questions and carry 1 mark each.
(iii) Q. no. 6 to 10 are short answer questions and carry 2 marks each.
(iv) Q. no. 11 to 22 are also short answer questions and carry 3 marks each.
(v) Q. no. 23 is a value based question and carries 4 marks.
(vi) Q. no. 24 to 26 are long answer questions and carry 5 marks each.
(vii) Use log tables if necessary, use of calculators is not allowed.

section-A a resultant of 28 N. If the angle of their inclination


1. Fifteen vectors, each of magnitude 5 units, are is 60, find the magnitude of each force.
represented by the sides of a closed polygon, all 7. The position of a particle is given by
taken in same order. What will be their resultant?
r = 3.0 ti 2.0 t 2 j + 4.0 k m
2. Can an object be accelerated without speeding up where t is in seconds and the coefficients have the
or slowing down? Explain. proper units for r to be in metres.

3. A projectile is projected at an angle of 15 to the (a) Find v and a of the particle?
horizontal with speed v. If another projectile is (b) What is the magnitude and direction of velocity
projected with the same speed, then at what angle of the particle at t = 2.0 s?
with the horizontal it must be projected so as to 8. A projectile is thrown with an initial velocity of
have the same range.
xi + y j. The range of the projectile is twice the
4. A wooden box is lying on an inclined plane. What y
is the coefficient of static friction if the box starts maximum height of the projectile. Calculate .
x
sliding when the angle of inclination is just 30?
OR
5. Two bodies of masses M and m are allowed to fall A light string passing over a smooth pulley connects
from the same height. If air resistance for each be two blocks of masses m1 and m2 (vertically). If the
the same, then will both the bodies reach the Earth acceleration of the system is g/8, find the ratio of the
simultaneously? two masses.
section-B 9. The driver of a truck travelling with a velocity v
6. Two forces whose magnitude are in the ratio 3 : 5 give suddenly notices a brick wall in front of him at a

24 Physics For you | AUGUST 16


distance d. Is it better for him to apply brakes or 20. A block A of mass 4 kg is placed on another block B
to make a circular turn without applying brakes in of mass 5 kg, and the block B rests on a smooth
order to just avoid crashing into a wall? Explain. horizontal table. For sliding the block A on B, a
10. A retarding force is applied to stop a motor car. If horizontal force of 12 N is required to be applied
the speed of the motor car is doubled, how much on it. How much maximum horizontal force can be
more distance will it cover before stopping under applied on B so that both A and B move together?
the same retarding force? Also find out the acceleration produced by this
section-c force.
11. On a certain day, rain was falling vertically with
a speed of 35 m s1. A wind started blowing after
sometime with a speed of 12 m s1 in east to west
direction. In which direction should a boy waiting
at a bus stop hold his umbrella? OR
12. Explain how a vector can be resolved into its A box of mass 4 kg rests upon an inclined plane.
rectangular components in three dimensions. The inclination of the plane to the horizontal is
13. Define scalar product of two vectors. Give its gradually increased. It is found that when the slope
geometrical interpretation. of the plane is 1 in 3, the box starts sliding down the
14. What do you mean by impulse of a force? Show that
plane. Given g = 9.8 m s2
impulse is equal to the product of average force and (i) Find the coefficient of friction between the
the time interval for which the force acts. Give the box and the plane.
units and dimensions of impulse. (ii) What force applied to the box parallel to the
plane will just make it move up the plane?
15. Show that there are two angles of projection for a
projectile to have the same horizontal range. What 21. Show that Newton's second law of motion is the
will be the maximum heights attained in the two real law of motion.
cases? Compare the two heights for q = 30 and 60. 22. A bird is sitting on the floor of a closed glass cage
16. A batsman deflects a ball by an angle of 45 and the cage is in the hand of a girl. Will the girl
without changing its initial speed which is equal to experience any change in the weight of the cage
54 km h1. What is the impulse imparted to the when the bird (i) starts flying in the cage with a
ball? (Mass of the ball is 0.15 kg.) constant velocity (ii) flies upwards with acceleration
17. Explain why (iii) flies downwards with acceleration?
(a) Porcelain objects are wrapped in paper or straw section-D
before packing for transportation?
23. Rahul was going to a nearby shopping complex
(b) Mountain roads are generally made winding
along with his father who was a Physics teacher.
upwards rather than going straight up?
Suddenly it started raining heavily. Rahul's father
18. A stone of mass 5 kg falls from the top of a cliff 50 m
advised him to hold umbrella slightly inclined
high and buries itself 1 m in sand. Find the average
with the vertical in the direction of motion. Out of
resistance offered by the sand and the time it takes
his curiosity, Rahul asked his father why should I
to penetrate.
hold umbrella slightly inclined even when the rain
19. Three equal weights A, B and C of drops are falling vertically downwards. His father
mass m each are hanging on a explained him the reason behind it satisfactorily.
string over a fixed pulley as shown (a) What are the values being displayed by Rahul?
in figure. (b) What are the values displayed by Rahul's
What are the tensions in the father?
strings connecting weights A to B (c) Give reason for holding the umbrella slightly
and B to C? inclined with the vertical.

Physics For you | AUGUST 16 25


section-e 3. To have the same range (with same speed), the
24. Define centripetal acceleration. Derive an expression sum of the two angles of projection should be 90.
for the centripetal acceleration of a body moving Since one of the angles is 15, the other should be
with uniform speed v along a circular path of radius 90 15 = 75.
r. Explain how it acts along the radius towards the 4. Here, angle of friction, q = 30
centre of the circular path. Coefficient of static friction,
OR ms = tan q = tan 30 = 0.5774
What is a projectile? Derive the expression for the 5. Net force acting on the body of mass M, i.e.,
trajectory, time of flight, maximum height and F = Mg f, f being the force of air resistance.
horizontal range for a projectile thrown upwards, Net acceleration acting on the body of mass M, i.e.,
making an angle q with the horizontal direction. F Mg f f
a= = =g
25. What is meant by banking of roads? What is the M M M
need for banking a road? Obtain an expression for Net acceleration acting on the body of mass m, i.e.,
the maximum speed with which a vehicle can safely f
a = g
negotiate a curved road banked at an angle q. The m
coefficient of friction between the wheels and the If M > m, then a > a. Hence the body of larger mass
road is m. will reach the Earth earlier.
OR 6. Let A and B be the two forces
(a) Obtain an expression for the acceleration of a Then A = 3x; B = 5x; R = 28 N and q = 60
body sliding down a rough inclined plane. Thus, A/B = 3/5
(b) Explain why it is easier to pull a lawn roller
than to push it? Now R = A2 + B2 + 2 AB cos q
26. (a) State the law of conservation of linear \ 28 = (3x )2 + (5x )2 + 2(3x )(5x )cos 60
momentum and derive it from Newton's second
law of motion. or 28 = 9 x 2 + 25x 2 + 15x 2 = 7 x or x = 28/7 = 4
(b) A disc of mass 10 g is kept floating horizontally \ Forces are; A = 3 4 = 12 N and B = 5 4 = 20 N
by throwing 10 marbles per second against it from 7. (a) Velocity,
below. If the mass of each marble is 5 g, calculate
dr d
the velocity with which the marbles are striking v= = (3.0 t i 2.0 t 2 j + 4.0 k )
dt dt
the disc. Assume that the marbles strike the disc
normally and rebound downward with the same = [3.0 i 4.0 t j] m s 1

speed. dv d
Acceleration, a = = (3.0 i 4.0 t j)
OR dt dt
(a) State the laws of limiting friction. Is kinetic = 0 4.0 j = 4.0 j m s 2
friction less than or greater than the coefficient of (b) At time t = 2 s,

static friction? v = 3.0 i 4.0 2 j = 3.0i 8.0 j
(b) Show that the tangent of the angle of friction is 2 2 1
equal to the coefficient of static friction. \ v = (3.0) + (8) = 73 = 8.54 m s
If q is the angle which makes with x-axis, then
solutions v y 8
1. Zero vector. The vector sum of all the vectors tan q = = = 2.667
vx 3
represented by the sides of a closed polygon taken
in the same order is zero. \ q = tan1(2.667) 70 with x-axis

2. Yes. For example, when a body moves along a 8. Here, range of projectile = 2 maximum height
circular path with constant speed, it possesses v 2 sin 2q v 2 sin2 q or 2v2 sin q cos q = v2 sin2q
=2
centripetal acceleration due to continuous change g 2g
in its direction of motion. or 2(v sin q)(v cos q) = (v sin q)(v sin q)

26 Physics For you | AUGUST 16


But v cos q = x and v sin q = y (given) The magnitude of the resultant velocity is
y v = v R2 + vW
2
= (35)2 + (12)2 = 37 m s 1
\ 2yx = y2 or 2x = y or = 2
x
Let resultant velocity v makes an angle q with the
OR vertical.
m m2 g m1 m2 AC vW
As a = 1 g \ = g Then, tan q = = = 0.343
m1 + m2 8 m1 + m2 OA v R
m1 m2 1 (m + m2 ) + (m1 m2 ) 8 + 1
or = or 1 = \ q = tan 1(0.343) 19
m1 + m2 8 (m1 + m2 ) (m1 m2 ) 8 1
Hence the boy should hold umbrella bending it
m1 9 towards east making an angle of about 19 with the
or = or m1 : m2 = 9 : 7
m2 7 vertical.

9. Suppose force FB is required in applying brakes to stop 12. Suppose vector A is represented by OP , as shown
the truck in distance d. Then in figure. Taking O as origin, construct a rectangular
mv 2 v2 parallelopiped with its three edges along the three
FB = ma = a =
2d 2d rectangular axes i.e., X-, Y- and Z-axis.
Suppose FT force is required in taking a turn of
radius d. Then
mv 2 1
FT = = 2FB or FB = FT
d 2
Clearly, it is better to apply brakes than to take a
circular turn.
10. Let F be retarding force applied on the motor car.
F
So retarding acceleration, a = = constant
m

Using kinematics equation, v2 = u2 + 2as Thus Ax , A y and Az are the three rectangular

u2 components of A.
Here v = 0, \ 0 = u2 + 2(a)s s =
2a Applying triangle law of vectors, OP = OT + TP
Now speed of the car is doubled so distance covered Applying parallelogram law of vectors,

by it will be s (say). OT = OR + OQ

u2 \ OP = OR + OQ + TP
\ 0 = (2u)2 + 2(a)s s = 4 = 4 s
2a But TP = OS
11. In figure,
Velocity of rain, Hence OP = OR + OQ + OS

or A = Ax + A y + Az = Ax i + A y j + Az k
v R = OA = 35 m s 1, vertically downward
If a, b and g are the angles between A and X-, Y-
and Z-axes repectively, then
Ax = A cos a, Ay = A cos b, Az = A cos g
We note that
OP 2 = OT 2 + TP 2 = OQ 2 + QT 2 + TP 2
or A2 = Ax2 + A2y + Az2 or A = Ax2 + A2y + Az2
13. The scalar or dot product of

two vector A and B is
defined as the product of
Velocity of wind,
the magnitudes of A and
vW = OB = 12 m s 1 , east to west
B and cosine of the angle q between them. Thus

Physics For you | AUGUST 16 27



A B = A B cos q = AB cos q t2 t2 t
J = Fav dt = Fav dt = Fav [t ]t2 = Fav (t2 t1 )
As A, B and cos q are all scalars, so the dot product 1
t1 t1
of A and B is a scalar quantity. Both A and B have
direction but their dot product has no direction. or J = Fav t , where t = t2 t1
Geometrical interpretation of scalar product : As Thus, the impulse of a force is equal to the product

shown in figure (a), suppose two vectors A and B of the average force and the time interval for which

are represented by OP and OQ respectively and it acts.
POQ = q. Dimensions of impulse = [MLT1]
SI unit of impulse = kg m s1
A B = AB cos q = A(B cos q) = A(OR )
= A Magnitude of component of B in the direction 15. The horizontal range of a projectile projected at an

of A angle q with the horizontal velocity u is given by
u2 sin 2q
R=
g
Replacing q by (90 q), we get
u2 sin 2(90 q) u2 sin(180 2q) u2 sin 2q
R = = =
g g g
i.e., R = R
From figure (b), we have Hence for a given velocity of projection, a projectile

A B = AB cos q = B( A cos q) = B(OS) has the same horizontal range of the angles of
projection q and (90 q). As shown in figure, the
= B Magnitude of component of A in the direction
horizontal range is maximum of 45. Clearly, R is
of B.
same for q = 15 and 75 but less than Rm. Again R
Thus the scalar product of two vectors is equal to
is same for q = 30 and 60.
the product of magnitude of one vector and the
magnitude of component of other vector in the
direction of first vector.
14. Impulse is defined as the product of the force and
the time for which it acts and is equal to the total
change in momentum.
Impulse = Force time duration
= Total change in momentum

Impulse is a vector quantity denoted by J . Its
direction is same as that of force. The impulse of Maximum height attained in first case,
a force is positive, negative or zero depending on u2 sin2 q
the momentum of the body increases, decreases or H1 =
2g
remains unchanged.
Maximum height attained in second case,
Suppose a force F acts for a small time dt. The
impulse of the force is given by u2 sin2 (90 q) u2 cos2 q
H2 = =
dJ = Fdt 2g 2g

If we consider a finite interval of time from t1 to t2, Again,


then the impulse will be H (30) u2 sin2 30 2g
=
t2 H (60) 2g u sin2 60
2
J = dJ = Fdt
t1 (1 / 2)2 1
= =
2 3
If Fav is the average force, then ( 3 / 2)

28 Physics For you | AUGUST 16


16. Let the ball of mass m prevents skidding. If road is straight up then q is
moving along AO large then f is small.
with initial speed u Due to this reason, mountain roads generally made
hits the bat PQ and winding upward rather than going straight up.
is deflected by the 18. Let v be the velocity acquired by stone after falling
batsman along OB through a height of 50 m.
(without change in the speed of the ball), such that
AOB = 45. Clearly, v = 2 gh = 2 9.8 50 = 980 m s 1
Let ON be the normal to the bat such that Now v is the velocity with which the stone starts
burying itself into the sand and finally comes to rest
45
AON = BON = = 22.5 in it after travelling a distance of 1 m.
2 From vf2 vi2 = 2as,
The initial momentum of the ball can be resolved
0 v2 = 2as (as vf = 0, vi = v)
into two components
(i) mu cos22.5 along NO and v2
or a= (neglecting negative sign)
(ii) mu sin22.5 along PQ 2s
Also, the final momentum of the ball can be resolved 980
or a = = 490 m s 2 (as s = 1 m)
into two components 2(1)
(i) mu cos22.5 along ON and Let F be the average resistance offered by the sand
(ii) mu sin22.5 along PQ to the stone. Since F acts vertically upwards and mg
The component of the momentum of the ball acts vertically downwards,
along PQ remains unchanged (both in magnitude F mg = ma or F = m(g + a)
and direction). However, the components of the = (5)(9.8 + 490) = 2499 N
momentum of the ball along ON are equal in Let t be the time taken by the stone to penetrate the
magnitude but opposite in direction. Since the sand.
impulse imparted by the batsman to the ball is equal From vf = vi + at ; 0 = 31.30 + (490)t
to the change in momentum of the ball along ON, 31.30
or t = = 0.06 s
Impulse = mucos22.5 (mucos22.5) 490
= 2 mucos22.5 19. Let us draw free-body diagrams for the weights A, B
Here, m = 0.15 kg ; u = 54 km h1 = 15 m s1 and C as shown in figure (a), (b) and (c).
Therefore, impulse = 2 0.15 15 cos 22.5
= 2 0.15 15 0.9239
= 4.16 kg m s1
17. (a) When porcelain objects are wrapped in paper
or straw, the time of impact between themselves is
very much increased during jerk in transportation.
P
F=
t
Hence, force on the porcelains is reduced during From figure (a), (b) and (c)
transportation, and saves them from breakage. T1 mg = ma ...(i)
(b) A small portion of the road is shown in the T2 + mg T1 = ma ...(ii)
figure. and mg T2 = ma ...(iii)
Friction on the track, Adding equations (i), (ii) and (iii),
f = mN = mmgcosq g
mg = 3ma or a =
3
As roads winding
From equation (i), T1 = mg + ma = mg + m(g/3)
upwards, q is small,
4
f is large, which = mg
3

Physics For you | AUGUST 16 29


From equation (iii), T2 = mg ma = mg m(g/3) In the absence of any external force,
2 F = 0 and ma = 0
= mg
3 As m 0, therefore, a = 0.
20. Here, m1 = 4 kg, m2 = 5 kg Thus there is no acceleration when no force is
Force applied on block A = 12 N applied. That is in the absence of any external force
This force must atleast be equal to the kinetic a body at rest will remain at rest and a body in
friction applied on A by B. uniform motion will continue to move uniformly
\ 12 = f k = mk R = mk m1 g along the same straight path. Hence first law of
or 12 = mk 4 g motion is contained in the second law.

12 3 dp
or mk = = Third law is contained in the second law : Let B
4g g dt
The block B is on a smooth surface. Hence to move be the resulting change of momentum of B. Let
A and B together, the force F that can be applied on
FAB be the force (reaction) exerted by B on A. Let
B is equal to the frictional forces applied on A by B
and applied on B by A. dpA
be the resulting change of momentum of A.
F = mk m1 g + mk m2 g = mk (m1 + m2)g dt
3 According to Newton's second law,
= (4 + 5) g = 27 N
dp dp
g FBA = B and FAB = A
dt dt
As this force moves both the blocks together on a

smooth table, so the acceleration produced is \ FBA + FAB = dpB + dpA = d ( p B + pA )
F 27 dt dt dt
a= = = 3 m s 2
m1 + m2 4 + 5 In the absence of external force, the rate of change
d
OR
of momentum must be zero i.e., ( pB + pA ) must
be zero. dt
1
Here m = 4 kg, sin q = , g = 9.8 m s 2 \ FBA + FAB = 0
3
(i) Various forces acting on the box are shown in or FBA = FAB or Action = Reaction
figure. When the box just begins to slide, the forces Hence third law of motion is contained in the
are in equilibrium. second.
f = mg sin q and R = mg cos q As both first and third laws of motion are contained
f mg sin q in the second law, we can say that Newton's second
m= = = tan q law is the real law of motion.
R mg cos q
1 1 22. In a closed cage, the inside air is bound with the cage.
= = = 0.35 (i) As the acceleration is zero, there is no change
32 12 8
in the weight of the cage.
(ii) When the block moves up the inclined plane, (ii) In this cage, the reaction R is given by
friction f acts down the plane. So minimum force R Mg = Ma or R = M(g + a)
needed to just move the box up the inclined plane Thus the cage will appear heavier than before.
F = mg sin q + f = mg sin q + mR (iii) In this case, the reaction R is given by
= mg(sin q + m cos q) [... R = mg cos q] Mg R = Ma or R = M(g a)
Thus the cage will appear lighter than before.
1 1 8
= 4 9. 8 + = 26.13 N 23. (a) Quest for knowledge
3 8 3 (b) Concern for the child and scientific thinking.
21. First law is contained in the second law : According (c) The child experiences the velocity of the rain
to Newton's second law of motion, the force acting relative to himself. To protect himself from the rain,
on a body is given by the child should hold umbrella in the direction of
F = ma relative velocity of rain with respect to the child.

30 Physics For you | AUGUST 16


24. An acceleration which is directed along the radius angle Dq also approaches zero. In this limit, as
towards the centre of the circular path is called AB = AC, so ABC = ACB = 90. Thus the change

centripetal acceleration. in velocity Dv and hence the acceleration a is

Consider a particle moving on a circular path of perpendicular to the velocity vector v1. But v1 is
radius r and centre O, with a uniform speed v as directed along tangent at point P, so acceleration

shown in figure (a). Suppose at time t the particle a acts along the radius towards the centre of the
is at P and at time t + Dt, the particle is at Q. Let circle.

v1 and v2 be the velocity vectors at P and Q. OR
Any object launched in an arbitrary direction in
space with an initial velocity and then allowed to
move under the influence of gravity alone is called
a projectile. Suppose a body is projected with initial
velocity u, making an angle q with the horizontal.
The velocity u has two rectangular components
(i) The horizontal component u cos q, which
remains constant throughout the motion.
(ii) The vertical component u sin q, which changes
with time under the effect of gravity.
Applying triangle law of vector addition in DABC, Y

AB + BC = AC
vy
\ BC = AC AB = v 2 v1
v A Path of
Thus the change in velocity in time Dt is given by x projectile
v
BC = Dv u sin u P(x, y) x
Also OP = OQ = r, radius of the circle. y Maximum
height = hm

v1 = v2 = v i.e., AB = AC = v vx = u cos
O u cos X
And POQ = Dq, BAC = Dq B
Thus the two triangles POQ and BAC are similar. R
PQ BC vy = u sin v
Hence, =
OP AB Equation of trajectory of a projectile : Suppose the
v
or Ds = Dv or Dv = Ds body reaches the point P(x, y) after time t.
r v r \ The horizontal distance covered by the body
Dividing both sides by Dt, we get in time t,
Dv v Ds x = Horizontal velocity time = u cos q t
= x
Dt r Dt or t =
u cos q
Taking the limit Dt 0 on both sides, we get
For vertical motion : u = u sin q, a = g, so the
Dv v Ds vertical distance covered in time t is given by
lim = lim
Dt 0 Dt r Dt 0 Dt 1
s = ut + at 2
Dv dv 2
But lim = = a, is the instantaneous acceleration x 1 x2
Dt 0 Dt dt or y = u sin q g.
Ds ds u cos q 2 u2 cos2 q
and lim = = v , is the instantaneous velocity. g
Dt 0 Dt dt or y = x tan q 2 x2
2
v v2 2u cos q
\ a = v or a = = w2r [... v = wr]
r r Thus y is a quadratic function of x. Hence the
This gives the magnitude of the acceleration of the trajectory of a projectile is a parabola.
particle in uniform circular motion. Time of flight : It is the time taken by the projectile
Direction of acceleration : As Dt tends to zero, the from the instant it is projected till it reaches a point

Physics For you | AUGUST 16 31


in the horizontal plane of its projection. The body Equating the forces along horizontal and vertical
reaches the point B after the time of flight Tf. direction respectively, we get
\ Net vertical displacement covered during the mv 2
time of flight = 0 R sin q + f cos q = ...(i)
r
1
As s = ut + at 2 mg + f sin q = R cos q, where f = mR
2 or R cos q f sin q = mg ...(ii)
1
\ 0 = u sinqT f gT f2 Dividing equation (i) by equation (ii), we get
2
R sin q + f cos q v 2
or T f = 2 u sin q =
R cos q f sin q rg
g
Maximum height of a projectile : It is the maximum Dividing numerator and denominator of LHS by
R cos q, we get
vertical distance attained by the projectile above the
horizontal plane of projection. f
tan q + 2 2
At the highest point A, vertical component of R = v or tan q + m = v
m =
f
f 1 m tan q rg R
velocity = 0 1 tan q rg
As v2 u2 = 2as R
u2 sin2 q 2 m + tan q m + tan q
\ 02 (u sin q)2 = 2 (g)hm or hm = or v = rg or v = rg . 1 m tan q ...(iii)
2g 1 m tan q
For m = 0 in equation (iii)
Horizontal range : It is the horizontal distance
v = (rg tan q)1/2
travelled by the projectile during its time of flight.
So, OR
Horizontal range = Horizontal velocity time of flight (a) Consider a body of weight mg placed on an
2u sin q u2 inclined plane. Suppose the angle of inclination q
or R = u cos q = 2 sin q cos q be greater than the angle of repose. Let a be the
g g
acceleration with which the body slides down the
u2 sin 2q inclined plane.
or R= [... 2 sin q cos q = sin 2 q]
g The weight mg has two rectangular components :
25. The process of raising the outer edge of a curved (i) mg cos q perpendicular to the inclined plane.
road above its inner edge through certain angle is It balances the normal reaction R. Thus
called banking of the road. R = mg cos q
(ii) mg sin q down the inclined plane.
Consider a car of weight mg going along a curved
If f k is the kinetic friction, then the net force acting
path of radius r with speed v on a road banked at an
down the plane is
angle q. The forces acting on the vehicle are
F = mg sin q f k
1. Weight mg acting vertically downwards
But f k = mk R = mk mg cos q
2. Normal reaction R of the road acting at an angle
\ ma = mg sin q mk mg cos q
q with the vertical. Hence a = g(sin q mk cos q)
3. Force of friction f acting downwards along the (b) As shown in figure (a), suppose a force F is
inclined plane. applied to pull a lawn roller of weight W. The force
F has two rectangular components
(i) Horizontal component F cos q helps to move
the roller foraward.
(ii) Vertical component F sin q acts in the upward
direction.
If R is the normal reaction, then
R + F sin q = W or R = W F sin q
Force of kinetic friction,
f k = mk R = mk(W F sin q) ...(i)

32 Physics For you | AUGUST 16


F R 20 5 103 v = 10 103 9.8
10 9.8
v= = 0.98 m s1 = 98 cm s1
100
OR
F cos
(a) Laws of limiting friction :

F sin F (i) The limiting friction depends on the nature of
f k
the surfaces in contact and their state of polish.
(ii) The limiting friction acts tangential to the two
W
(b) surfaces in contact and in a direction opposite to
As shown in figure (b), if a force F is applied to push the direction of motion of the body.
a roller of weight W, then the normal reaction is (iii) The value of limiting friction is independent
R = W + F sin q of the area of the surface in contact so long as the
Force of kinetic friction, normal reaction remains the same.
f k = mk R = mk (W + F sin q) ...(ii) (iv) The limiting friction (fs)max is directly
Comparing (i) and (ii) we find that f k > f k proportional to the normal reaction R between the
i.e., the force of friction is more in case of push than two surfaces.
in case of pull. So it is easier to pull a lawn roller i.e., ( f s )max R or ( f s )max = m s R
than to push it.
( f s )max Limiting friction
26. (a) In the absence of any external force, the vector or m s = =
R Normal reaction
sum of the linear momentum of a number of The proportionality constant ms
particles in an isolated system remains constant and is called coefficient of static
remains unchanged by their mutual interaction.
friction. It is defined as the
Consider an isolated system of n particles. Suppose
ratio of limiting friction to the
the n particles have masses m1, m2, m3, ..., mn and are
normal reaction.
moving with velocities v1 , v2 , v3 ,..., vn , respectively.
Then total linear momentum of the system is As f k < ( f s )max or mk R < m s R \ mk < m s
Thus the coefficient of kinetic friction is less than
p = m1v1 + m2v2 + m3v3 + .... + mnvn
the coefficient of static friction.
= p1 + p2 + p3 + ... + pn
(b) W is the weight of the body, R is the normal
If F is the external force acting on the system, then reaction, (fs)max is the limiting friction, F is the
dp
according to Newton's second law, F = applied force and OC is the resultant of (fs)max and
dt R.

For an isolated system, F = 0 or dp = 0 C B
dt
As the derivative of a constant is zero, so R

p = constant or p1 + p2 + p3 + .... + pn = constant
Thus in the absence of any external force, the total Applied force
A F
linear momentum of the system is constant. (fs)max O
(b) Mass of each marble piece,
m = 5 g = 5 103 kg
W
Number of marbles thrown per second = 10
Let velocity of impact of each marble = v BC OA ( f s )max
\ tan q = = =
Change in momentum of each marble OB OB R
= mv ( mv) = 2 mv ( f s )max
But = m s = Coefficient of static friction
Change in momentum per second = 2 mv 10 R
\ Force exerted by marbles on the disc = 20 mv \ tan q = ms
But the disc can be kept floating if this force balances Thus the coefficient of static friction is equal to the
the weight of the disc. tangent of the angle of friction.
\ 20 mv = Mg

Physics For you | AUGUST 16 33


Class XI

T his specially designed column enables students to self analyse their


extent of understanding of specified chapters. Give yourself four
marks for correct answer and deduct one mark for wrong answer.
Self check table given at the end will help you to check your
readiness.

Kinematics
Total Marks : 120 Time Taken : 60 min

NEET / AiiMs / PMTs


(c) u 1 + 3 cos2 q (d) ucosq
Only One Option Correct Type 2
1. Points P,Q and R are in a vertical line such that 5. A man running at a speed of 5 km h1 finds that
PQ = QR. A ball at P is allowed to fall freely. the rain falls vertically. When he stops running, he
The ratio of times of descent through PQ and QR is finds that the rain is falling at an angle of 60 with
(a) 1 : 1 (b) 1 : 2 the horizontal. The velocity of rain with respect to
running man is
(c) 1 : ( 2 1) (d) 1 : ( 2 + 1)
5 5 3
2. Two particles A and B are moving in x-y plane. (a) km h 1 (b) km h 1
Particle A moves along a line with equation y = x while 3 2
B moves along x-axis such that their x-coordinates 4 3
(c) km h 1 (d) 5 3 km h 1
are always equal. If B moves with a uniform speed 5
3 m s1, the speed of A is 6. A certain vector in the x-y plane has an x component
3 of 4 m and a y component of 10 m. It is then rotated
(a) 3 2 m s 1 (b) m s 1
2 in the x-y plane so that its x component is doubled.
1 Then its new y component is
(c) m s 1 (d) 3 m s1
3 (a) 20 m (b) 7.2 m
3. Two cars are moving in the same direction with a (c) 5.0 m (d) 4.5 m
speed of 30 km h1. They are separated from each 7. A police party is moving in a jeep at a constant
other by 5 km. Third car moving in the opposite speed v. They saw a thief at a distance x on a
direction meets the two cars after an interval of motorcycle which is at rest. At the same moment
4 minutes. The speed of the third car is the thief saw the police and he started at constant
(a) 30 km h1 (b) 35 km h1 acceleration a. Which of the following relations
1
(c) 40 km h (d) 45 km h1 is true, if the police is able to catch the thief?
4. A particle is projected from the ground with an (a) v2 < ax (b) v2 < 2ax
initial speed of u at an angle of projection q with (c) v2 2ax (d) v2 = ax
horizontal. The average velocity of the particle 8. A car starts from rest and accelerates at uniform rate
between its time of projection and time it reaches of 6 m s2 for some time, then moves with constant
highest point of trajectory is speed for some time and retards at the same uniform
u u rate and comes to rest. Total time for the journey
(a) 1 + 2 cos2 q (b) 1 + 2 sin2 q
2 2 is 24 s and average speed for journey is 20 m s1.

34 Physics For you | AUGUST 16


How long does the car move with constant speed? 13. Assertion : Two balls of different masses are
(a) 4 s (b) 8 s thrown vertically upward with same speed. They
(c) 12 s (d) 16 s will pass through their point of projection in the
9. A particle P is moving in a circle of radius R with downward direction with the same speed.
uniform speed v. C is the centre of the circle and AB Reason : The maximum height and downward
is a diameter. The angular velocity of P about A and velocity attained at the point of projection are
C are in the ratio independent of the mass of the ball.
(a) 1 : 1 (b) 1 : 2 14. Assertion : A body can have acceleration even if its
(c) 2 : 1 (d) 4 : 1 velocity is zero at a given instant of time.
10. The distance travelled by a particle in time t is given Reason : The body is momentarily at rest when it
by x = kt3, where k = 10 m s3. The average speed of reverses it direction of motion.
the particle from t = 1 s to t = 5 s is 15. Assertion : A particle in x-y plane is related by
(a) 280 m s1 (b) 300 m s1 x = a sin wt and y = a(1 cos wt), where a and w
(c) 310 m s1 (d) 320 m s1 are constants, then the particle will have parabolic
11. The maximum height attained by a projectile motion.
is increased by 1% by increasing its speed of Reason : A particle under the influence of two
projection without changing the angle of projection. perpendicular velocities has parabolic motion.
Then the percentage increase in the horizontal
JEE MAiN / JEE AdvANcEd / PETs
range will be
Only One Option Correct Type
(a) 2% (b) 1% (c) 0.5% (d) 0.2%
16. A particle moves rectilinearly. Its displacement x at
12. Given below are four curves describing variation of
time t is given by x2 = at2 + b, where a and b are
velocity with time of a particle. Which one of these
constants. Its acceleration at time t is proportional to
describe the motion of a particle initially in positive
direction with constant negative acceleration? (a) 1 (b)
1 1
3 x x2
x
1 t
(c) (d)
2
x x2
17. Bamboo strips are hinged to form three rhombi
as shown. Point A0 is fixed to a rigid support.
The lengths of the sides of the rhombi are in the
ratio 3 : 2 : 1. Point A3 is pulled with a speed v.
Let vA1 and vA2 be the speeds with which points
A1 and A2 move. Then, the ratio vA1 : vA2 is

(a) W (b) X (c) Y (d) Z


Assertion & Reason Type
Directions : In the following questions, a statement of
assertion (A) is followed by a statement of reason (R). Mark
the correct choice as : (a) 2 : 3 (b) 3 : 5 (c) 3 : 2 (d) 5 : 3
(a) If both assertion and reason are true and reason is the
18. A point P moves in counter-clockwise direction on a
correct explanation of assertion. circular path as shown in the figure. The movement
(b) If both assertion and reason are true but reason is not of P is such that it sweeps out a length s = t3 + 5,
the correct explanation of assertion. where s is in metres and t is in seconds. The radius
(c) If assertion is true but reason is false. of the path of 20 m. The acceleration of P when
(d) If both assertion and reason are false. t = 2 s is nearly

Physics For you | AUGUST 16 35


(b) the displacement of the bolt during the free fall
relative to the elevator shaft is 0.75 m.
(c) the distance covered by the bolt during the free
fall relative to the elevator shaft is 1.38 m.
(d) the distance covered by the bolt during the free
fall relative to the elevator shaft is 2.52 m.
(a) 14 m s2 (b) 13 m s2
23. The magnitude of the vector product of two vectors
(c) 12 m s2 (d) 7.2 m s2
A and B may be
19. Two NCC cadets simultaneously aim their guns at a (a) greater than AB (b) equal to AB
target put on a tower. The first cadet fired the bullet (c) less than AB (d) equal to zero
with a speed of 100 m s1 at an angle of 30 with
Integer Answer Type
the horizontal. The second cadet, ahead of the first
cadet by a distance of 50 m, fired the bullet with a 24. A racing car is travelling along a track at a constant
speed of 80 m s1 at an angle of q with the horizontal. speed of 40 m s1. A TV cameraman is recording the
Both the bullets hit the target simultaneously. event from a distance of 30 m directly away from the
Which of the following is correct? track as shown in figure. In order to keep the car under
5 5 view, what should be angular speed of the camera
(a) sin q = (b) cos q = (in rad s1) when q is 30?
8 6
40 m s1
5 3
(c) tan q = (d) tan q =
3 5
30 m


More than One Options Correct Type
20. The distances covered by a freely falling body in its
first, second, third, ... nth seconds of its motion 25. A particle moving in a straight line covers half
(a) forms an arithmetic progression the distance with speed 3 m s1. The other half
(b) form a series corresponding to the squares of of the distance is covered in two equal intervals
the first n natural numbers with speeds 4.5 m s1 and 7.5 m s1 respectively.
(c) do not form any well defined series The average speed of the particle (in m s1) during
(d) forms a series corresponding to the difference of this motion is
the squares of the successive natural numbers.
21. Velocity of a particle moving in a curvilinear path

varies with time as v = (2t i + t 2 j) m s 1 . Here, t is
in second. At t = 1 s,
(a) acceleration of particle is 8 m s2.
6
(b) tangential acceleration of particle is m s 2 .
5
2
(c) radial acceleration of particle is m s 2.
5
5 5
(d) radius of curvature to the path is m.
2
22. An elevator ascends with an upward acceleration
of 2.0 m s2. At the instant its upward speed is
2.5 m s1, a loose bolt is dropped from the ceiling of
the elevator 3.0 m from the floor. If g = 10 m s2, then
(a) the time of the flight of the bolt from the ceiling
to floor of the elevator is 0.71 s. 0124-6601200 for further assistance.

36 Physics For you | AUGUST 16


26. A particle is projected at an angle of 60 above the
(B) A C = B (Q)
horizontal with a speed of 10 m s1. After some
time, the direction of its velocity makes an angle of
30 above the horizontal. The speed of the particle
2n
at this instant is m s 1 . What is the value of n?
3
(C) B A = C (R)
Comprehension Type
A balloon of mass 100 kg with a basket of mass 25 kg
attached to it is descending with a uniform downward
velocity 8 m s1. A person of mass 50 kg is also standing

in the basket. He throws a packet of mass 2 kg with an (D) A + B + C = 0 (S)
initial velocity 6 m s1 relative to him and perpendicular
to the motion of the descending balloon. The person
observes that the packet hits the ground 4 s after being
thrown. Assume that the balloon continues to move A B C D
with the same uniform velocity vertically downward (a) P R Q S
and g = 10 m s2. (b) R Q S P
27. Height of the balloon when the packet hits the (c) S R P Q
ground is nearly (d) Q P R S
(a) 92 m (b) 72 m (c) 80 m (d) 112 m 30. Four balls of same masses are projected with the
28. Velocity of the packet 1.6 s after being thrown, equal speeds at angles 1x5, 30, 60 and 75 with
the horizontal direction from the ground. Match
relative to the person in the basket, has a magnitude
the quantities of Column I with the angle of projec-
nearly equal to tion given in Column II.
(a) 17 m s1 (b) 12 m s1 Column I Column II
(c) 21 m s1 (d) 10 m s1
(A) Horizontal range is (P) 15
Matrix Match Type
maximum for angle
29. Given below in Column I are the relations between (B) The vertical height is (Q) 30

vectors A, B and C and in Column II are the maximum for angle
(C) The time of flight is (R) 60
orientations of A, B and C in the x-y plane. Match
the relations in Column I to correct orientations in minimum for angle
Column II. (D) The horizontal range is (S) 75
minimum for angle
Column I Column II A B C D
(a) P, R P S Q, R
(A) A + B = C (P)
(b) Q, R S P P, S
(c) R, S Q R P, Q
(d) P, S R Q R, S
Keys are published in this issue. Search now! J

Check your score! If your score is


> 90% ExcEllENT work ! You are well prepared to take the challenge of final exam.

No. of questions attempted 90-75% Good work ! You can score good in the final exam.
No. of questions correct 74-60% sATisFAcTory ! You need to score more next time
Marks scored in percentage < 60% NoT sATisFAcTory! Revise thoroughly and strengthen your concepts.

Physics For you | AUGUST 16 37


If A is not normal to I
F Electric current and current Density
Electric current is the rate of flow of electric charge but makes an angle q
through a cross section of a conductor with the normal to
dq current, then
I= I = JA cos q
dt
I
J=
A cos q
Current density is a vector quantity and its SI unit
is A m2.
q
For steady current, I = If a steady current flows in a
t metallic conductor of non
DQ
For average current, Iav = uniform cross section.
Dt
Electric current is a scalar quantity because it does f Along the wire, current
not obey the law of vectors. Its SI unit is ampere is same, I1 = I2
(A) or coulomb per second (C s1) f Current density depends inversely on area, so
One ampere of current means the flow of 6.25 1018 J1 > J2, as A1 < A2
electrons per second through any cross section of
conductor.
F Thermal Velocity and Drift Velocity
Thermal velocities of free electrons are randomly
Current density at a point in a conductor is the
distributed in all possible direction of a metallic
ratio of the current at that point in the conductor
conductor.
to the area of cross section of the conductor of
that point, Average thermal velocity is zero but average thermal
dI speed is non zero.
J= n or, I = J dA Drift velocity is defined as the velocity with which
dA

38 Physics For you | AUGUST 16


the free electrons get The first two coloured rings from the left end
drifted towards the indicate the first two significant figures of the
positive terminal under resistance in ohms. The third colour ring indicates
the effect of the applied the decimal multiplier and the last colour ring
electric field. stands for the tolerance in percent.

eE
f vd = a t vd = t
m
f Order of drift velocity is 104 m s1.
f t is the relaxation time which is average time
elapsed between two successive collisions. It The colour code of a resistor is as shown in the
decreases with increase in temperature. table.
F relation between current Density, conductivity Colour Number Multiplier Tolerance
and Electric Field Black 0 100
Let the number of free electrons per unit volume Brown 1 101
in a conductor = n Red 2 102
Total number of electrons in dx distance = n(Adx) Orange 3 103
Total charge dQ = n(Adx)e Yellow 4 104
Cross sectional area = A Green 5 105
Blue 6 106
Violet 7 107
Gray 8 108
White 9 109
Gold 101 5%
Silver 102 10%
dQ dx No colour 20%
Current I = = nAe I = neAvd
dt dt
Current density J = = nevd J = ne eE t
I F resistance and its combinations
A m When a potential difference is applied across
ne 2 t a conductor, free electrons get accelerated and
J = E J = sE collide with positive ions and their motion is thus
m
opposed. This opposition offered by the ions is
ne 2 t called resistance of the conductor.
Conductivity s =
m Resistance is the property of a conductor by virtue

In vector form J = sE (Another form of Ohm's law) of which it opposes the flow of current in it.
s depends only on the material of the conductor rl rl 2 rV
and its temperature. R= = =
A V A2
I 1V rl Resistance of the conductor depends on the
Ohm's law, J = sE = V = I = RI
A r l A temperature and varies as Rt = R 0 (1 + a Dt),
f At a given temperature, current is directly where Rt = Resistance at t C, R0 = resistance at
proportional to the applied potential difference. 0 C, Dt = change in temperature, a = temperature
f The substances which obey ohm's law are coefficient of resistance.
called ohmic. For metal : a is positive and for semiconductors
F colour code of resistors and insulators : a is negative.
A colour code is used to indicate the resistance Resistance of the conductor decreases linearly
value and its percentage accuracy. with decrease in temperature and becomes zero at

Physics For you | AUGUST 16 39


a specific temperature. This temperature is called f
critical temperature, at this temperature conductor
becomes a superconductor.

Resistance between square faces


distance between faces a
RAB = r =r
area of square b2
Resistance between rectangular faces
b r
RXY = r = (does not depend on b)
a b a
A frame of cube is made with
wires each of equal resistance r
then
f Resistance between two nearer
7
corners R12 = r
12
3
f Resistance across face diagonal R13 = r
4
5
f Resistance across main diagonal R17 = r
6
If a wire is stretched to n times of its original length,
F EMF ( e ) and Terminal Voltage ( V )
The potential difference across the terminals of a
its new resistance will be n2 times.
cell when it is not producing any current is called
If a wire is stretched such that its radius is reduced
emf of the cell.
1
to of its original values, then resistance The energy given by the cell in the flow of unit
nth
charge in the whole circuit (including the cell) is
will increase n 4 times similarly resistance will
called the emf of the cell.
decrease n4 time if radius is increased n times by
emf depends on
contraction.
Resistance of different shaped conductors. f nature of electrolyte
f metal of electrodes
emf does not depend on
f area of plates
f distance between the electrodes
f quantity of eletrolyte
f size of cell
When current drawn through the cell or current
is supplied to cell, the potential difference across
its terminals = terminal voltage.
f When current I is drawn
from the cell, then terminal
voltage is less than its
emf.
V = e Ir

40 Physics For you | AUGUST 16


f At the time of charging a cell, current is supplied f Resistance offered by the electrolyte of the cell
to the cell, the terminal voltage is greater than when the electric current flows through it is
emf e. known as internal resistance.
V = e + Ir f Distance between two electrodes increases
Efficiency of cell is defined by r increases
Total potential difference f Area dipped in electrolyte increases
= 100%
emf r decreases
Internal resistance of a cell f Concentration of electrolyte increases
r decreases
f Temperature increases r decreases

f The closed loop can be traversed in any


F Kirchhoff's Laws direction. While traversing a loop if potential
First law (Kirchhoff 's current law)
increases, put a positive sign in expression and
f In an electric circuit, the algebraic sum of the if potential decreases put a negative sign.
current meeting at any junction in the circuit
+ V1
is zero.
+
I = 0 V4 V2
+
f From the figure, + V3
I1 I2 I3 I4 + I5 = 0
I1 + I5 = I2 + I3 + I4 f From the figure, V1 V2 + V3 V4 = 0. Boxes
may contain resistor or battery or any other
f This is based on law of element (linear or nonlinear).
conservation of charge.
f This law is based on conservation of energy.
Second law (Kirchhoff 's voltage law) F Wheatstone Bridge
f The algebraic sum of all the potential differences It consists of four resistances R1, R2, R3 and R4
along a closed loop is zero. which are connected to form a quadrilateral ABCD
SIR + SEMF = 0 as shown in figure.

Physics For you | AUGUST 16 41


f The experiment should be repeated by
interchanging the positions of X and R to
minimize an error due to contact resistance.
F Potentiometer
Potentiometer is an ideal voltmeter because it
draws no current from the circuit at the instant
of measurement.
Working : It is based on the fact that the fall of
In a balanced condition even though current flows potential across any portion of the wire is directly
in the rest of the circuit, galvanometer will not proportional to the length of that portion provided
show any deflection (i.e. Ig = 0). the wire is of uniform area of cross-section and a
R R constant current is flowing through it.
Also, 1 = 3 i.e., V l (If I and A are constants.)
R2 R4
or V = kl
The measurement of resistance by Wheatstone's
where k is known as potential gradient i.e., fall of
bridge is not affected by the internal resistance
potential per unit length of the given wire.
of the cell.
Circuit of potentiometer
F Meter-Bridge
Meterbridge is the modification of Wheatstone's
network used to determine unknown resistance.
It is an instrument which works on the principle
of Wheatstone's network. So it is also called
Wheatstone's meterbridge.
The length of wire used is one metre, so it is called f Primary circuit contains constant source of
meterbridge. voltage, rheostat of resistance box and resistance
of the potentiometer of wire.
f Secondary circuit contains unknown circuit
with galvanometer.
Let r = Resistance per unit length of potentiometer
wire
Application of Potentiometer
f Comparision of emf of two cells
(i) plug only in (1 2)
Jockey is at position J
From the balancing condition, balancing length AJ = l1
X Resistance of wire AD of length lx e1 = kl1
= (ii) Plug only in (2 3)
R Resistance of wire DC of length lr
Jockey is at position J
lx balancing length AJ = l2
X = R
100 lx e2 = kl2
e1 l1
Minimization of errors in measurement of X \ =
f The wire used must be uniform i.e., of same e2 l2
cross section. f Internal resistance of given primary cell
The value of R should be so chosen that the e V
f e = V + Ir r =
null point is obtained as near (close) to the I
centre of wire as possible. e V
or r = R
V

42 Physics For you | AUGUST 16


Key K open e = kl1 (AJ = l1) 1 hp = 746 watt.
Key K closed total potential difference If one heater boils a certain mass of water in time
V = kl2 (AJ = l2) t1 and another heater boils the same mass of water
l1 l2 in time t2, then both the heaters are connected in
r = R series, the same water will boil in time (t1 + t2);
l2
when both the heaters are connected in parallel the
t1 t2
same water will boil in time t = .
t1 + t2
If P1, P2, P3 .... are the powers of electric appliances
in series with source of rated voltage V, the effective
power consumed is
1 1 1 1
= + + + ......
PS P1 P2 P3
If P1, P2, P3 are the powers of electric appliances
in parallel with a source of rated voltage V, the
f The emf of battery (driver cell or auxiliary cell) effective power consumed is
must be greater than emf 's to be compared i.e., PP = P1 + P2 + P3 + .....
e > e1, e > e2 and for the combination method
e > (e1 + e2)... F charging and Discharging of capacitors
Charging a capacitor
f The positive terminal of e 1 or e 2 or of the
f Charge on the capacitor at any time t,
combination must be connected to that end of
potentiometer wire where positive terminal of q = CV(1 e t/RC)
the battery (driving cell) is connected. f Current in the circuit at any
f The potentiometer wire must be uniform. time t,
V
f The resistance of potentiometer wire should I = e(t /RC )
be high. R
f Maximum charge on capacitor, q0 = CV.
F Joule's Law of heating f Maximum current in the circuit, I0 = V/R.
According to Joules heating effect of current, the f RC is known as time constant (tC) of circuit.
amount of heat produced H in a conductor of
f VC + VR = V ...(i)
resistance R, carrying current I for time t is
H = I2Rt (in joule) f Charging means q increases. So if q increase
I 2 Rt (in calorie)
then VC increases so from equation (i) V R
or H = decreases and hence I decreases.
J
where J is Joules mechanical equivalent of heat
(= 4.2 J cal1).
F Electric Power
It is defined as the rate at which work is done by
the source of emf in maintaining the current in
the electric circuit.
electric work done
Electric power P =
time taken
V2
P = VI = I 2 R = .
R
The SI unit of power is watt (W).
The practical unit of power is kilowatt (kW) and
horse power (hp).
1 kilowatt = 1000 watt.

Physics For you | AUGUST 16 43


Discharging a capacitor
f Charge on the capacitor at any time t,
q = q0 et/RC ;
q0 = CV = initial charge on the capacitor at t = 0
f Current during discharging
dq
I= = I0e t /RC
dt f In discharing all the parameters (q, VC, VR, I)
decay exponentially.
V
I0 = = initial current at t = 0 In both charging and discharging current decreases
R
exponentially i.e. capacitor blocks D.C.
f VC + VR = 0
In time t = RC = tC (time constant) any parameter
|VC| = |VR| ...(i) changes by 63%.
f Discharging means q decreases so VC decreases. If time constant (tC) of a circuit is very less, then
Hence from eqn (i) |VR| decreases and initial changes in any parameter (q, VC, VR, I) is
correspondingly I decreases. so rapid.

1. An electron moves in a circle of radius 10 cm with 1 4V 8V 2


P Q
a constant speed of 4.0 106 m s1. The electric r1 r2
current at a point on the circle will be
(a) 2.0 1012 A (b) 1.0 1012 A
(c) 3.2 1012 A (d) 6.4 1012 A 9
R
2. A metal rod of length 10 cm and a rectangular
1 1 A and 3 V
cross-section of 1 cm cm is connected to a (a) (b) 1 A and 4 V
2 3 6
battery across opposite faces. The resistance will be 1 A and 9 V 1 A and 12 V
(a) maximum when the battery is connected (b) (d)
9 12
1 4. n batteries are connected to form a circuit as shown
across 1 cm cm faces.
2 in the figure. The resistances denote the internal
(b) maximum when the battery is connected resistances of the batteries which are related to the
across 10 cm 1 cm faces. emf s as ri = kei where k is a constant with proper SI
(c) maximum when the battery is connected units. The solid dots represent the terminals of the
1 batteries. Then
across 10 cm cm faces. rn
2 1 r1 2 r2 3 r3 n
(d) same irrespective of the three faces.
3. Two batteries of emf 4 V and 8 V with internal
resistance 1 W and 2 W are connected in a circuit n
(a) the current through the circuit is .
with resistance of 9 W as shown in figure. The k
current and potential difference between the points (b) the potential difference between the terminals
P and Q are of the ith battery is zero.

44 Physics For you | AUGUST 16


n2 The total resistance of the network between points
(c) the current through the circuit is . A and B is equal to
k
(d) the potential difference between the terminals (a) R1 + R12 + 2R1R2 (b) R1 R12 + R1R2
e
of the ith battery is .
k (c) R2 + R22 + 2R1R2 (d) R2 R22 + R1R2
5. Two non-ideal batteries are connected in parallel.
Consider the following statements : 10. The equivalent resistance (RAB) between the point
(A) The equivalent emf is smaller than either of A and B is
C
the two emfs.
(B) The equivalent internal resistance is smaller 2 4

than either of the two internal resistances. 3


A B
Which one is correct regarding the given statements?
(a) Both A and B are correct. 6 12
(b) A is correct but B is wrong. D
(c) B is correct but A is wrong.
(d) Both A and B are wrong.
6. A metal wire of diameter 2 mm and length 100 m
(a) 6 W (b) 7.5 W
has a resistance of 0.5475 W at 20 C and 0.805 W at
(c) 4.5 W (d) 8 W
150 C. Then
(a) the temperature coefficient of resistance is 11. A fully charged capacitor C discharges through a
3.6 103 C1 resistor R. After how many time constants does the
(b) resistance at 0C is 0.5107 W stored energy drop to half its initial value?
(c) resistivities at 20C is 1.72 108 W m (a) 0.20 (b) 0.02
(d) All of above (c) 0.35 (d) 0.16
7. The current in a conductor varies with time t as 12. Refer to the circuit shown in the figure.
I = 2t + 3t2, where I is in ampere and t in seconds. 3 A 2 B 2
Electric charge flowing through a section of the 9V 8 8 4
conductor during t =2 s to t = 3 s is
(a) 10 C (b) 24 C (c) 33 C (d) 44 C 2 D 2 C 2

8. I-V characteristics of a copper wire of length L and The current through the
area of cross-section A is shown in figure. The slope (a) 3 W resistor is 0.50 A
of the curve becomes (b) 3 W resistor is 0.25 A
(a) more if the experiment (c) 4 W resistor is 0.50 A
is performed at higher (d) 4 W resistor is 0.25 A
I
temperature 13. In a large building, there are 15 bulbs of 40 W,
(b) more if a wire of steel of V 5 bulbs of 100 W, 5 fans of 80 W and 1 heater of 1 kW.
same dimension is used The voltage of the electric mains is 220 V. The
(c) more if the length of the wire is increased minimum capacity of the main fuse of the building
(d) less if the length of the wire is increased will be
9. A network of resistors of resistances R1 and R2 (a) 14 A (b) 8 A
extends off to infinity to the right as shown in the (c) 10 A (d) 12 A
figure. [JEE Main 2014]
A R1 C R1 R1 14. Two metal wires of identical dimensions
R2 R2 R2 are connected in series. If s1 and s2 are the
conductivities of the metal wires respectively, the
B R1 D R1 R1 effective conductivity of the combination is

Physics For you | AUGUST 16 45


s1 + s2 s1s2 The fall of potential per km is 8 V and the average
(a) (b) resistance per km is 0.5 W. The power loss in the
s1s2 s1 + s2
wire is
2s1s2 s1 + s2 (a) 19.2 W (b) 19.2 kW
(c) (d)
s1 + s2 2s1s2 (c) 19.2 J (d) 12.2 kW
[AIPMT 2015] [AIPMT 2014]
15. A potentiometer wire of length L and a resistance 20. The resistances in the
r are connected in series with a battery of e.m.f. E0 two arms of the meter
5 R
and a resistance r1. An unknown e.m.f. E is balanced bridge are 5 W and R W
at a length l of the potentiometer wire. The e.m.f. E respectively. When the
will be given by resistance R is shunted with G

E0 l LE0r an equal resistance, the new A l 100 l1


B
1
(a) (b) balance point is at 1.6l1. The
L (r + r1 )l
resistance R is
LE0r E0 r l (a) 10 W (b) 15 W
(c) (d) .
lr1 (r + r1 ) L (c) 20 W (d) 25 W
[AIPMT 2014]
[AIPMT 2015]
16. When 5 V potential difference is applied SolutionS
across a wire of length 0.1 m, the drift speed of 1. (b) : Consider a point A on the circle. The electron
electrons is 2.5 104 m s1. If the electron density crosses this point once in every revolution. In
in the wire is 8 1028 m3, the resistivity of the one revolution, the electron travels 2p (10 cm)
material is close to distance. Hence, the number of revolutions made
(a) 1.6 106 W m (b) 1.6 105 W m by the electron in one second is
(c) 1.6 108 W m (d) 1.6 107 W m 4.0 106 2
[JEE Main 2015] 2
= 107.
20p 10 p
17. In the circuit shown, the The charge crossing the point A per second is
resistance r is a variable 2 3. 2
resistance. If for r = f R, the heat 107 1.6 1019 C = 1012 C.
p p
generation in r is maximum
then the value of f is Thus, the electric current at this point is
3. 2
(a)
1
(b) 1 1012 A 1.0 1012 A
2 p
1 3 l
(c) (d) 4 2. (a) : Since R = r , resistance will be maximum
4 A
[JEE Main Online 2016] when l is maximum and A is least. This holds good
for option (a) only.
18. The charge flowing through a resistance R varies
with time t as Q = at bt2, where a and b are positive 84 4 1
3. (a) : I = = = A
constants. The total heat produced in R is 1 + 2 + 9 12 3
a3 R 3 1
(a) (b) a R VP VQ = 4 3 = 3 V
2b b 3
a3 R a3 R 4. (b) : Suppose the current is I in the indicated
(c) (d) direction. Applying Kirchoff s loop law,
6b 3b
[NEET Phase-1 2016] e1 Ir1 + e2 Ir2 + e3 Ir3 + ... + en Irn = 0
19. Two cities are 150 km apart. Electric power is sent e1 + e2 + e3 + ... + en
or, I=
from one city to another city through copper wires. r1 + r2 + r3 + ...rn

48 Physics For you | AUGUST 16


e + e + e + ... + en 1 A A R1 C
= 1 2 3 = RT R2 RT
k(e1 + e2 + e3 + ...en ) k
B B R1 D
The potential difference between the terminals of
R2 RT
the ith battery is Clearly, RT = 2R1 +
R2 + RT
[ R2 || RT ]
1
ei Iri = ei (kei ) = 0 or
k RT2 2R1RT 2R1R2 = 0
5. (c) or RT = R1 R12 + 2R1R2
6. (d) : Here r = 1 mm = 103 m, l = 100 m,
Since RT > 0, RT = R1 + R12 + 2R1R2
t1 = 20 C, R1 = 0.5475 W, t2 = 150 C, R2 = 0.805 W
Temperature coefficient of resistance is 2W 6W
10. (c) : As =
R R1 0.805 0.5475 4 W 12 W
a= 2 = = 3.6 10 3C 1 . Hence, the Wheatstone bridge is balanced.
R1 (t 2 t1 ) 0.5475(150 20)
RACB = 2 + 4 = 6 W
Resistance at 0C, RADB = 6 + 12 = 18 W
R1 0.5475 0.5475 6 18
R0 = = 3
= = 0.5107 W RAB = = 4. 5 W
1 + at1 1 + 3.6 10 20 1.072 6 + 18
Resistivity at 0C, 11. (c) : In discharging the capacitor the charge on the
R A R pr 2 0.5107 3.14 (103 )2 capacitor varies as
r0 = 0 = 0 = q = q0et/t
l l 100
The energy stored in the capacitor is
= 1.60 108 W m
q 2 q02 2t /t
Resistivity at 20 C, U= = e = U 0 e 2t /t
r20 = r0 (1 + a t) 2C 2C
= 1.60 108 (1 + 3.6 103 20) in which U0 is the initial stored energy. The time at
= 1.60 108 1.072 = 1.72 108 W m 1
which U = U 0 is found from
7. (b) : Given I = 2t + 3t2 2
dq 1
As I = \ dq = Idt = (2t + 3t2)dt U 0 = U 0e 2t /t or 2 = e 2t /t
dt 2
Charge passed from t = 2 s to t = 3 s is ln 2
ln 2 = 2t/t or t =t = 0.35t
3 2
q = dq = (2t + 3t 2 )dt = [t 2 ]32 + [t 3 ]32 88
12. (d) : RBC (right hand side) = W
2
8+8
= (9 4) + (27 8) = 24 C
= 4 W (as 2 W + 4 W + 2 W = 8 W)
DI 1 RAD (right hand side) is again 4 W.
8. (d) : Slope of I V graph = =
DV R Equivalent resistance of the circuit,
If the experiment is performed at a higher R=3W+4W+2W=9W
temperature, the resistance R of copper increases V 9
Current drawn from battery I = = = 1 A
and hence slope decreases, so the option (a) is wrong. R 9
In options (b) and (c), the resistances increase and At A, I is equally divided (I/2) between 8 W resistance
so their slope become less. and the remaining circuit of 8 W. At B, (I/2) is
In option (d), the resistance R increases and so slope equally divided (I/4) between the 8 W resistor and
decreases. Hence only option (d) is correct. the remaining circuit of resistance 8 W.
9. (a) : Since the network is infinite, the resistance of Thus, current through 4 W resistor is I/4, i.e., 0.25 A.
the network to the right of points C and D is also 13. (d) : Power of 15 bulbs of 40 W = 15 40 = 600 W
equal to RT (net resistance). We can redraw the Power of 5 bulbs of 100 W = 5 100 = 500 W
circuit as show in figure. Power of 5 fans of 80 W = 5 80 = 400 W

Physics For you | AUGUST 16 49


Power of 1 heater of 1 kW = 1000 W The current through the potentiometer wire is
\ Total power, P = 600 + 500 + 400 + 1000 E0
I=
= 2500 W (r + r1 )
When these combination of bulbs, fans and heater
and the potential difference across the wire is
are connected to 220 V mains, current in the main
Er
fuse of building is given by V = Ir = 0
P 2500 (r + r1 )
I= = = 11.36 A 12 A The potential gradient along the potentiometer
V 220
wire is
14. (c) : As both metal wires are of identical dimensions,
V E0 r
so their length and area of cross-section will be k= =
same. Let them be l and A respectively. Then L (r + r1 )L
The resistance of the first wire is As the unknown e.m.f. E is balanced against length
l l of the potentiometer wire,
R1 = ... (i)
s1 A Er l
\ E = kl = 0
and that of the second wire is (r + r1 ) L
l 16. (b) : V = IR
R2 = ... (ii)
s2 A rl
As I = neAvd and R =
A
rl V
\ V = neAvd or r =
A ne vd l
As they are connected in series, so their effective
resistance is Here, V = 5 V, n = 8 1028 m3, vd = 2.5 10 4 m s1,
Rs = R1 + R2 l = 0.1 m, e = 1.6 1019 C
5
l l \ r= 19
= + (using (i) and (ii)) 28
8 10 1.6 10 2.5 10 4 0.1
s1 A s2 A
= 0.156 104 W m 1.6 105 W m
l 1 1 ... (iii)
= + 17. (a) : Let the source voltage be V.
A s1 s2
Equivalent resistance of the circuit when r = fR
If seff is the effective conductivity of the
rR fR ( 2 f + 1)R
combination, then Req = R + =R+ =
r+R f +1 ( f + 1)
2l ... (iv)
Rs = \ Current in the circuit, I = V = V ( f + 1)
seff A Req R( 2 f + 1)
Equating eqns. (iii) and (iv), we get Current in the resistance r(= fR)
2l l 1 1
= + I2 =
I
=
V
seff A A s1 s2 f + 1 R( 2 f + 1)
2 s + s1 2s s Now, heat generated per unit time in r
= 2 or s eff = 1 2
s eff s1s 2 s1 + s 2
V2 f
H = I 22 r =
15. (d) : E0 r1 R( 2 f + 1)2

For maximum H , dH = 0
L df
V2 1 4f
2
3
=0
l
R ( 2 f + 1) ( 2 f + 1)
E 1
G
or 2f + 1 4f = 0 f =
2

50 Physics For you | AUGUST 16


18. (c) : Given, Q = at bt2 20. (b) : In the first case,
dQ At balance point
\ I= = a 2bt
dt 5 l1
At t = 0, Q = 0 I = 0 = ...(i)
R 100 l1
Also, I = 0 at t = a/2b
()
\ Total heat produced in resistance R,
a /2b a /2b 5 R
2 2
H= I Rdt = R (a 2bt ) dt
0 0
a /2b G
2 2 2
= R (a + 4b t 4abt )dt A
l1 100 l1
B
0
a /2b
3 2
= R a2t + 4b2 t 4ab t In the second case, ()
3 2 0 At balance point R

1.6l1 5 R
5
a 4b2 a3 4ab a2 = ...(ii)
= R a 2 + (R / 2) 100 1.6l1
2b 3 8b3 2 4b2
Divide eqn. (i) by eqn. G
a3 R 1 1 1 a3 R (ii), we get
= + = A B
b 2 6 2 6b 1.6l1 100 1.6l1
19. (b) : Here, 1 100 1.6l1
=
Distance between two cities = 150 km 2 1.6(100 l1 )
Resistance of the wire, R = (0.5 W km1)(150 km) 160 1.6l1 = 200 3.2l1
= 75 W 40
Voltage drop across the wire, 1.6l1 = 40 or l1 = = 25 cm
1. 6
V = (8 V km1)(150 km) = 1200 V
Substituting this value in eqn. (i), we get
Power loss in the wire is
2 5 25 375
V 2 (1200 V) = or R= W = 15 W
P= = = 19200 W = 19.2 kW R 75 25
R 75 W

Mathematics Today 2015 ` 325


Mathematics Today 2014 ` 325
Mathematics Today 2013 ` 300
Chemistry Today 2010 ` 300
Physics For You 2010 ` 300
Biology Today 2015 ` 325
Add ` 60 as postal charges
April, May, June issues not available
Mathematics Today 2015 ` 240

Chemistry Today 2015 ` 240

Physics For You 2015 ` 240

Biology Today 2015 ` 240

Physics For you | AUGUST 16 51


CLASS XII Series 3

CBSE
Moving Charges and Magnetism Time Allowed : 3 hours
Magnetism and Matter Maximum Marks : 70
GENERAL INSTRUCTIONS Previous Years Analysis
(i) All questions are compulsory. 2016 2015 2014
(ii) Q. no. 1 to 5 are very short answer questions and carry 1 mark each. Delhi AI Delhi AI Delhi AI
(iii) Q. no. 6 to 10 are short answer questions and carry 2 marks each. VSA 1 1 2 1
(iv) Q. no. 11 to 22 are also short answer questions and carry 3 marks
SA-I 1
each.
SA-II 2 1 1 1 1
(v) Q. no. 23 is a value based question and carries 4 marks.
VBQ _ _ _ _ _ _
(vi) Q. no. 24 to 26 are long answer questions and carry 5 marks each.
LA _ 1 1 _ 1
(vii) Use log tables if necessary, use of calculators is not allowed.

section-A section-B
1. Give two points to compare the magnetic properties 6. Do magnetic forces obey Newtons third law. Verify
^
of steel and soft iron. for two current elements dl1 = dl i located at the
^
2. Two identical charged particles moving with the origin and dl2 = dl j located at (0, R, 0). Both carry
same speed enter a region of uniform magnetic current I.
field. If one of them enters normal to the field 7. A long wire is bent into a circular coil of one turn
direction and the other enters in a direction at and then into a circular coil of smaller radius having
30 with the field, what would be the ratio of their n turns. If the same current passes in both the cases,
angular frequencies? find the ratio of the magnetic fields produced at the
centres in the two cases.
3. What is the function of radial magnetic field in a
moving coil galvanometer? 8. A magnetising field of 1500 A m1 produces a
magnetic flux of 2.4 105 Wb in a bar of iron of
4. A solenoid coil of 300 turns m1 is carrying a current cross section 0.5 cm2. Calculate permeability and
of 5 A. The length of the solenoid is 0.5 m and has a susceptibility of the iron bar used.
radius of 1 cm. Find the magnitude of the magnetic OR
field inside the solenoid.
Out of the two magnetic materials, A has relative
5. In a hydrogen atom, the electron moves in an permeability slightly greater than unity while B has
orbit of radius 0.5 , making 1016 rps. Calculate less than unity. Identify the nature of the materials
the magnetic moment associated with the relative A and B. Will their susceptibilities be positive or
motion of electron. negative?

52 Physics For you | AUGUST 16


9. Using the relation for potential energy of a current 15. An electron moves around the nucleus in a hydrogen
carrying planar loop, in a uniform magnetic field, atom of radius 0.51 , with a velocity of 2.0 105 m s1.
obtain the expression for the work done in moving Calculate
the planar loop from its unstable equilibrium (a) the equivalent current due to orbital motion of
position to stable equilibrium position. electrons,
10. A beam of alpha particles and of protons having the (b) the magnetic field produced at the centre of the

same velocity enters a uniform magnetic field B at right nucleus, and
angles to the direction of field. The particles describe (c) the magnetic moment associated with the
circular paths. What is the ratio of the radii of circular electrons.
paths described by alpha particles and protons.
16. A galvanometer having 30 divisions has a current
section-c sensitivity of 20 mA per division. It has a resistance
11. Define current sensitivity and voltage sensitivity of of 20 W. How will you convert it into an ammeter
a galvanometer. Increase in the current sensitivity measuring upto 1 ampere? How will you convert
may not necessarily increase the voltage sensitivity this ammeter into voltmeter reading upto 1 V?
of a galvanometer. Justify.
17. Figure shows a long straight
12. The following figure shows the variation of intensity I
wire of a circular cross-
of magnetisation versus the applied magnetic field
section with radius a carrying
intensity, H, for two magnetic materials A and B. r1
steady current I. The current P
(a) Identify the materials A
I is uniformly distributed r2
and B. Q
across this cross-section.
(b) For the material A, plot
Calculate the magnetic field a
the variation of intensity
in the region r1 < a and r2 > a
of magnetisation versus I
and show its variation with
temperature.
distance from the axis of wire.
13. A compass needle free to turn in a horizontal plane
is placed at the centre of circular coil of 30 turns and 18. Equal currents are flowing through two infinitely
radius 12 cm. The coil is in a vertical plane making long parallel wires. Will there be a magnetic field at
an angle of 45 with the magnetic meridian. When a point exactly halfway between the wires when the
the current in the coil is 0.35 A, the needle points currents in them are
west to east. (a) in the same direction, and
(a) Determine the horizontal component of the (b) in opposite direction?
Earth's magnetic field at the location.
19. A combination of two long straight wires AB and DE
(b) The current in the coil is reversed , and the coil
and a semicircular arc BCD of radius R is shown in
is rotated about its vertical axis by an angle of
the figure. If a current I flows as shown in the figure,
90 in the anticlockwise sense looking from the
then find the net magnetic field at point O.
above. Predict the direction of the needle. Take
the magnetic declination at the places to be zero.
14. The following figure shows a part of an electric
circuit. PQRS is a rectangular loop of uniform
conducting wire. The lengths PQ and SR are quite
long but PS = QR = 2.0 cm. If a current of 10 A is
20. Two parallel coaxial circular coils of equal radius R
recorded by the ammeter A, find the magnetic force
and equal number of turns N carry equal current I
per unit length between the wires PQ and SR.
in the same direction and are separated by a distance
2R as shown in the figure. Find the magnitude and
direction of the net magnetic field produced at the
Is the magnetic force repulsive or attractive ? mid-point O of the line joining their centres.

Physics For you | AUGUST 16 53


about this, he decided to take help of his family,
friends and neighbours and arranged for the cost.
He convinced his uncle to undergo this test so as
to enable the doctor to diagnose the disease. He got
the test done and the resulting information greatly
OR helped the doctor to give him proper treatment.
An electron beam passes through a magnetic field of Based on the above paragraph, answer the
2.0 mT and an electric field of 3.4 104 V m1 both following:
acting simultaneously in mutually perpendicular (a) What according to you, are the values
directions. If the path of electrons remains undeflected, displayed by Sumit and his family, friends and
calculate the speed of the electrons. neighbours?
If the electric field is switched off, what will be the (b) Assuming that the MRI scan of his uncle's chest
was done by using a magnetic field of 1.5 T, find
radius of the electron path?
the maximum and minimum values of force
Given that me = 9.1 1031 kg and e = 1.60 1019 C.
that this magnetic field could exert on a proton
21. The figure shows the path of an electron that passes (charge = 1.6 1019 C) moving with a speed of
through two regions containing uniform magnetic 104 m s1. State the condition under which the
fields of magnitudes B1 and B2. The electron force can be minimum.
describes a semicircular path in each region.
section-e
(a) Which magnetic field is stronger?
24. (a) Using Biot-Savarts law, derive the expression
(b) What are the directions of fields B1 and B2 ?
for the magnetic field in the vector form at a point
(c) Is the time spent by the electron in region B1
on the axis of a circular current loop.
greater than, less than or the same as the time (b) What does a toroid consist of ? Find out the

spent in region B2 ? expression for the magnetic field inside a

B1
toroid for N turns of the coil having the average
e
radius r and carrying a current I. Show that the

magnetic field in the open space inside and
B2 exterior to the toroid is zero.
OR
(a) Using Amperes circuital law, obtain an
expression for the magnetic field along the axis
22. Two identical short magnetic dipoles P and Q of of a current carrying solenoid of length l and
having N number of turns.
magnetic moment m each, are placed at a separation
(b) A toroid has a core (non-ferromagnetic) of
of d with their axes mutually perpendicular to each
inner radius 19 cm and outer radius 21 cm,
other as shown in figure. Find the magnetic field at
around which 3000 turns of a wire are wound.
the point O midway between the dipoles.
If the current in the wire is 10 A, then find the
Q
P N magnetic field (i) outside the toroid, (ii) inside
O S the core of the toroid, and (iii) in the empty
S N space surrounded by the toroid.
d
25. (a) Deduce an expression for the frequency of
section-D revolution of a charged particle in a magnetic
23. Sumit's uncle was advised by his doctor to undergo field and show that it is independent of velocity
an MRI scan test of his chest and gave him an or energy of the particle.
estimate of the cost. Not knowing much about (b) Draw a schematic sketch of a cyclotron. Explain,
the significance of this test and finding it to be too giving the essential details of its construction, how
expensive he first hesitated. When Sumit learnt it is used to accelerate the charged particles.

54 Physics For you | AUGUST 16


OR m0 = 4p 107 T m A1
Distinguish the magnetic properties of dia-, So, B = m0nI = 4p 107 300 5
para- and ferro-magnetic substances in terms of = 1.9 103 T
(i) susceptibility, (ii) magnetic permeability and 5. Here, r = 0.5 = 0.5 1010 m, e = 1.6 1019C
(iii) coercivity. Give one example of each of these e
As m = IA = pr 2 = epr 2
materials. T
Draw the field lines due to an external magnetic = 1.6 10 1016 3.14 (0.5 1010)2 A m2
19

field near a (i) diamagnetic, (ii) paramagnetic = 1.256 1023 A m2


substance.
6. Given situation is shown in the figure.
26. (a) Draw a labelled diagram of a moving coil The magnetic field due to
galvanometer. Describe briefly its principle and
current element dl2 does
working.
not affect current element
(b) Which one of the two, an ammeter or a
dl1 . Hence, force on
milliammeter, has a higher resistance and
why? dl1 due to dl2 is zero.
The magnetic field due to
OR
(a) A small compass needle of magnetic moment current element dl1 affects current element dl2 .

m is free to turn about an axis perpendicular Hence force on dl2 due to dl1 is non-zero. Hence,
to the direction of uniform magnetic field B. the magnetic forces do not obey Newtons third
The moment of inertia of the needle about the law.
axis is I. The needle is slightly disturbed from 7. Let l be the length of the wire. When the wire is first
its stable position and then released. Prove that bent in the form of one turn circular coil,
it executes simple harmonic motion. Hence l
l = 2 pr1 , or r1 = , N =1
deduce the expression for its time period. 2p
(b) A compass needle whose magnetic moment m NI m 1 I m 0 pI
\ B1 = 0 = 0 =
is 60 A m2 pointing geographical north at a 2r1 2 (l / 2 p) l
certain place where the horizontal component When the wire is bent in the form of n-turns coil,
of earth's magnetic field is 40 mWb m2 l
experiences a torque of 1.2 103 N m. What is l = n 2 pr2 or r2 = ,N =n
2np
the declination of the place? solutions m NI m 0nI m pn 2 I
\ B2 = 0 = = 0
solutions 2r2 2(l / 2np) l
1. (i) Steel has high retentivity and soft iron has low B1 1
So, = or, B1 : B2 = 1 : n2
retentivity. B2 n2
(ii) Hysteresis loss is more in case of steel than in 8. Here, H = 1500 A m1, f = 2.4 105 Wb,
case of soft iron. A = 0.5 cm2 = 0.5 104 m2
qB Magnetic induction,
2. Angular frequency, w = . It is independent of
m f 2.4 105
B= = 4
= 0.48 Wb m 2
angle q. For same q, B and m, w remains the same, A 0.5 10
B 0.48
i.e., the ratio of the angular frequencies of the two Permeability, m = = = 3.2 104 T m A 1
charged particles is w1 : w2 = 1 : 1. H 1500
As m = m0 (1 + cm)
3. Radial magnetic field provides constant magnetic m
\ Susceptibility, cm = 1
field and hence the torque on the coil becomes m0
constant in all positions of the coil for the given 3.2 104
current. This provides a linear current scale. = 1
4 3.14 107
4. Here, n = 300 turns m1, I = 5 A,
= 254.77 1 = 253.77

Physics For you | AUGUST 16 55


OR Therefore, the increase in current sensitivity of
Relative permeability, mr = 1 + cm galvanometer may not necessarily increase the
\ cm = mr 1 voltage sensitivity of the galvanometer.
As the relative permeability of A is slightly greater If n 2n then R 2R.
than 1, so its susceptibility c is small and positive. Hence, Is = 2Is
Hence A is a paramagnetic substance. and Vs = Vs .
As the relative permeability of B is slightly less than 12. (a) Slope of I-H graph gives susceptibility
1, so its susceptibility c is small and negative. I
Hence B is a diamagnetic substance. cm = of the material.
H
9. Potential energy of current loop when its dipole For material A, the slope is positive and smaller, it is

moment m makes angle q with the field B is given by likely to be paramagnetic.
U = mB cos q For material B, the slope is positive and larger, it is
In unstable equilibrium position, q = 180 likely to be ferromagnetic.
\ U1 = mB cos 180 = +mB (b) For the paramagnetic material A,
In stable equilibrium position, q = 0 B
\ U2 = mB cos0 = mB I =C 0
T
Required work, B0 = applied magnetic field
W = U2 U1 = mB mB
\ I T = const
= 2mB = 2IAB [Q m = IA]
This represents a rectangular hyperbola.
10. We know that radius of circular path described by
a charged particle moving with a speed v at right 13. (a) As per question, number of turns in the coil,
N = 30, radius of the coil, R = 12 cm = 0.12 m and
angle to uniform magnetic field B is given by
ma q p current flowing in the coil, I = 0.35 A
mv r
r= a = \ Magnetic field at the centre of the coil,
qB rp m p qa m NI
B= 0
But we know that, ma = 4mp and qa = 2 qp 2R
ra 4m p q p 2 4 p 107 30 0.35
= = or ra = 2rp B= = 5.5 105 T
rp m p 2q p 1 2 0.12
11. Current sensitivity of a galvanometer is defined as As the coil is in a vertical
the deflection produced in the galvanometer when plane making an angle
of 45 with the magnetic
a unit current flows through it.
meridian, hence magnetic
Voltage sensitivity of a galvanometer is defined as
the deflection produced in the galvanometer when field B is in horizontal
a unit voltage is applied across the two terminals of plane making an angle

the galvanometer. 45 east of south. B can
Let q be the deflection produced in the galvanometer be resolved into two
on applying voltage V, then components
q nBA (i) B cos 45 due south, and
Current sensitivity, I s = = (ii) B sin 45 due east.
I k
As the compass needle points west to east, it is clear
q nBA
Voltage sensitivity, Vs = = that B cos 45 due south is just balanced by BH due
V kR north and net field in horizontal plane is B sin 45
Thus, the current sensitivity can be increased due east.
by increasing, n, B, A and by decreasing k. If \ BH = B cos 45
n is increased, it will increase the resistance of 1
= 5.5 105 = 3.9 105 T
conductor. 2
The voltage sensitivity can be increased by increasing (b) When direction of current in the coil is
n, B, A and by decreasing k and R. reversed and the coil is rotated about its vertical

56 Physics For you | AUGUST 16


axis by an angle of 90 in the anticlockwise sense, The resistance of the ammeter formed is

the direction of field B will be in horizontal plane R g Rs 20 0.012
making an angle of 45 west of south. Thus, it can Rg = = = 0.012 W
R g + Rs 20 + 0.012
be easily shown that compass needle points east to
west. Current for full scale deflection, I g = 1 A
Voltage range, V = 1 V
14. Here, current flowing through each wire of PQ and SR,
\ Required series resistance,
10
I= = 5 A , d = 2.0 cm = 2.0 102 m V 1
2 R= Rg = 0.012 = 0.988 W
I g 1
\ Force per unit length between the wires
PQ and SR, 17. The current I is uniformly distributed in the wire,
2 7 I
m 2I 10 2 5 5 current per unit area can be calculated as .
F= 0 = pa2
4p d (2 102 ) (a) To calculate magnetic field at P, let us consider
a loop of radius r1, and apply Amperes law.
= 2.5 104 N m1
By right hand thumb rule the force between the B . dl = m0Inet
wires PQ and SR is attractive in nature. I
B 2pr1 = m0 pr 2
11 2 1
15. Radius of electron orbit, R = 0.51 = 5.1 10 m; pa
velocity of electron, v = 2.0 105 m s1; mass of m0 Ir1
B=
electron, m = 9.1 1031 kg and charge on electron, 2pa2
e = 1.6 1019 C B r1
(a) Equivalent current due to orbital motion of (b) To calculate magnetic field at Q, let us consider
electron, a loop of radius r2, and apply Amperes law.

ev 1.6 1019 2.0 105
I= = = 1 104 A = m0Inet
B. dl
2 pR 2 3.14 5.1 1011 B
B 2pr2 = m0I
(b) Magnetic field at the centre of the nucleus, m I
B= 0
m I 4 p 107 1 104 2p r2 Br
B 1/ r
B= 0 = 1
2R 2 5.1 1011 B
r2 r
= 1.23 T
(c) Magnetic moment associated with the electron, A graph can be plotted showing variation of magnetic
field B with distance r from the axis of wire.
| m | = IA = IpR2
= 1 104 3.14 (5.1 1011)2 18. Let there be two infinitely long parallel wires
= 8.2 1025 A m2 carrying a current I each and separation between
16. Here n = 30, Rg = 20 W then is d.
Current sensitivity, k = 20 mA per division
\ Current required for full-scale deflection is
Ig = nk = 30 20 = 600 mA = 6 104 A = 0.0006 A
(i) For conversion into ammeter, I = 1 A
Ig 0.0006 20
\ RS = Rg = = 0.012 W
I Ig 1 0.0006

i.e., a shunt of 0.012 W should be connected


(a) If both wires carry current in same direction
across the galvanometer.
as shown in figure (a), then the magnetic field
(ii) For conversion of resulting ammeter into at mid-point due to both wires have same
voltmeter. magnitude,

Physics For you | AUGUST 16 57


m0 I m I OR
BP = BQ = = 0 31
Here, me = 9.1 10 kg,
d pd
2p e = 1.6 1019 C, B = 2.0 mT = 2.0 103 T
2
and E = 3.4 104 V m1
But, BP and BQ are in opposite directions.
As electron beam goes undeflected,
Hence, net magnetic field at the mid-point
between the two wires, E 3.4 104
Hence, v = = = 1.7 107 m s1
m I m I B 2.0 103
B = BP + BQ = 0 + 0 = 0
pd pd If the electric field is switched off, then under
(b) If two wires carry current in mutually opposite the magnetic field, electron beam will describe a
directions as shown in figure (b), then the net circular path of radius,
magnetic field at the mid-point between the mv 9.1 1031 1.7 107
two wires, r= e =
eB 1.6 1019 2.0 103
m I m I 2m I
B = BP + BQ = 0 + 0 = 0 , = 4.8 102 m = 4.8 cm
pd pd pd
which is finite and nonzero. 21. (a) We know that radius of circular path of a charge
mv
19. Let B1 , B2 and B3 be the magnetic fields at in a magnetic field is given by r =
point O due to current carrying straight wire qB
AB, semicircular wire BCD and straight wire DE In the given figure, radius of semicircular path in

respectively. region of B1 is less than that in region B2 , hence it
m I is obvious that B1 > B2.
Then, B1 = 0 (b) Applying Fleming's left-hand rule, we find that
4 pR
[Since point O lies near one end of long wire at a B1 is directed into the plane of paper but B2 is
normal distance R] directed out of the plane of paper.
m I m I (c) Time of revolution of a charged particle in a
B2 = 0 and B3 = 0
4R 4 pR 2pm
magnetic field, T =
\ Net magnetic field at O is given by qB
m I m I m I Hence, time to complete a semicircular path,
B = B1 + B2 + B3 = 0 + 0 + 0
4 pR 4 R 4 p R T pm
t= =
m I 2 qB
| B | = 0 [2 + p]
4 pR As mass and charge of electron are constant,

and B is directed perpendicular to plane of paper t B
Hence, 1 = 2
directed out of it. t2 B1
20. We know that the magnetic field at a point on the axial As B1 > B2, hence we conclude that t1 < t2. So,
line of a circular current carrying coil is given by
the time spent by the electron in region B1 is

m 0 NIR 2 less than that in region B2 .
B=
2(R 2 + x 2 )3/ 2 d
22. The point O lies at a distance, r = from the centre
Here, x = R. As both coils are identical and carry 2
of either magnet (short). For magnet P, the point O
current in the same direction, hence
lies on axial line but for magnet Q, the point O lies
m 0 NIR 2 m NIR 2 m NI on the equatorial line.
B1 = B2 = = 0 = 0
2 2 3/ 2 3 4 2R
2(R + R ) 4 2R m 2m m 0 2m m 16m
Hence, | BP | = 0 = = 0
4p r 3 4p d 3 4 p d3
Directions of B1 and B2 are same along the axial line
towards the right side.
2
m NI m0 m m0 m m 8m
Hence, B = B1 + B2 = B1 + B1 = 2B1 = 0 and | BQ | = = = 0
2 2R 4 p r 3 4 p d 3 4 p d 3
along the direction A1OA2.
2

58 Physics For you | AUGUST 16



Directions of BP and BQ are shown in figure. into it. Hence, current enclosed is zero.
\ B=0
OR
(a) Refer to point 3.2 (2) page no. 171 (MTG Excel
in Physics)
19 + 21
(b) Mean radius r = = 20 cm
Obviously, the magnitude of resultant magnetic 2
3000
field B will be Number of turns per unit length =
2p(20 102 )
| B | = BP2 + BQ2 = 2388.5 turns m1
m 8m 2 5 m 0m (i) Outside the toroid, B = 0
= 0 (2)2 + (1)2 = (ii) Inside the toroid, B = m0nI
3
4p d
pd 3
The magnetic field B subtends an angle b from the = 4p 107 2388.5 10
axes joining two magnets P and Q, where B = 0.03 T
(iii) In the empty space surrounded by toroid, B = 0
m0 8 m
BQ 4 p d3 1 25. (a) Refer to point 3.3 (2) page no. 172(MTG Excel
tan b = = = = 0.50 in Physics)
BP m 0 16 m 2
4 p d3 (b) Refer to point 3.3 (5) page no. 173 (MTG Excel
1
b = tan (0.50) = 26.6 in Physics)
23. (a) (i) Presence of mind ; OR
High degree of general awareness; Refer to point 3.8 (8) page no. 180, 181 (MTG Excel
Ability to take prompt decisions ; in Physics)
Concern for his uncle ; 26. (a) Refer to point 3.4 (2) page no. 175 (MTG Excel
(ii) Empathy ; Helping and caring nature. in Physics)
(b) Maximum force I Rg
(b) Shunt resistance, Rs = g
= qvB = 1.6 1019 104 1.5 = 2.4 1015 N I Ig

Force is maximum when v is perpendicular to B .
Clearly, the shunt needed to convert
Minimum force = 0
galvanometer into a milliammeter has a larger
Force is minimum when v is parallel to B . value than that required to convert into an
24. (a) Refer to point 3 (vi) page no. 170 (MTG Excel ammeter. As the shunt resistance is connected
in Physics) in parallel with the galvanometer, so the
(b) A long solenoid shaped in the form of closed milliammeter will have a higher resistance than
ring is called a toroid. According to Amperes's the ammeter.
circuital law OR

B . dl = m0 I (a) Refer to point 3.6 (7) page no. 179 (MTG Excel
in Physics)
\ B2pr = m0NI
(b) In stable equilibrium, a compass needle points
B = m0nI along magnetic north and experiences no torque.
N When it is turned through declination a, it
where n = = number of turns per unit length points along geographic north and experiences
2pr
torque, t = mB sin a
Inside : The current enclosed by the loop, in the
t 1.2 103 1
open space inside the toroid, is zero. Hence B = 0. \ sin a = =
6
=
mB 60 40 10 2
Exterior : The current coming out of the plane of
the paper is cancelled exactly by the current going or a = 30

60 Physics For you | AUGUST 16


C RE
C NCEPT Electrostatic Potential Energy and
Grounding of Conductors
When we keep two like charges close to each other, Fext is almost equal and opposite to Fe since we are to
due to repulsion they move away from each other and move the charge q slowly (quasi statically) so our applied
gain kinetic energy. force should be such that it appears to be at almost
How do we explain this? equilibrium at all instant but over a prolonged period,
One way is we say that the repulsive forces pushes them the charge would appear to have been shifted.
away from each other due to which they accelerate dr = ds cosq is the displacement along the radial vector,
and gain energy. i.e., displacement in the direction of force.
Another way is energy method. We know that energy \ Work done by external agent,
can neither be created, nor be destroyed in an isolated Wext = dwext = (Fext )(dr )
system. So considering the two charges together as a r2
kqQ 1
system, initially from a zero KE (kinetic energy) they = Fe dr = 2
dr ; where k =
attain non-zero KE, which means there has been a r1
r 4 0

gain of KE, therefore there has to be loss of some other r


1 kqQ kqQ
2

form of energy, which is the potential energy here. It = kqQ \ Wext =


is a stored form of energy which depends upon the r r1 r2 r1
relative positions of the charges. This work clearly is only initial and final positions
Mathematically, PE (Potential energy) of any given dependent but independent of path. Hence we conclude
system of charges is defined as the minimum amount that the external agent was performing work against
of work that is required to be done to build the given the conservative electrostatic forces.
system, beginning from a situation from where they can Therefore we can associate PE with it, which by
be assumed to be placed for away from each other. definition is,
Let us first calculate the work required to be done in Wext = DPE
moving one charge q in the vicinity of another charge kqQ kqQ
Q from a position A to position B in the path shown \ U2 U1 =
r2 r1
in the figure. \ PE at a separation r becomes,
A s kqQ
ds co U(r) = + U where U indicates the potential
dr = Fe
r1
r q r
Fext ds energy at infinite separation, that is conventionally
taken to be zero until and unless specified.
Q
r2 B So, if U = 0 then
ds is a small instantaneous displacement along the kqQ
U (r ) =
path. r
Fe and Fext are the electrostatic and external forces 1
respectively. \ U , so can we conclude that by increasing
r
Contributed By: Bishwajit Barnwal, Aakash Institute, Kolkata
Physics For you | AUGUST 16 61
separation, PE decreases? No, not if the charges are Let us see few examples of it before moving to next
kqQ section.
unlike. If qQ < 0, then U = < 0, so increasing r,
r E1. Consider two point charges of given charges q1 and
gives a less negative PE, so obviously PE increases. q2 and masses m1 and m2 which are released from
Forget this trap of ve and +ve. Let me give you a an initial separation r0. Find their relative velocity
Golden rule! when the separation between them becomes large.
Whenever you release any system and allow them to
q1 r0 q2
move spontaneously, it always moves towards lower v1 v2
Sol.:
PE configuration.
q1 q2
So, if q and Q are both +ve or both ve, due to repulsion
they would spontaneously move away which mean if Notice that on combined mass system, (there are
r increases, PE decreases. no external forces, so linear momentum has to be
If q and Q are unlike, they attract and move closer so conserved which means, m1v1 = m2v2 = p (say)
if r decrease, PE decreases, hence if r increases, PE \ Using energy conservation (DE = 0),
increases.
KEi + PEi = KEf + PEf
Similarly, if we use the definition of PE as work done
by external agent to build the system assuming they kq1q2 1 1 p2 p2
= m1v12 + m2v22 = +
were initially placed far away. r0 2 2 2m1 2m2
r1 = (far away) kq1q2 p2 1 1 p2
r2 = r = + =
r0 2 m1 m2 2
\ Wext = kqQ kqQ = kqQ = PE at a separation r m1m2
r r where = = reduced mass of the system
m1 + m2
PE of a System of three Point charges
Assume, that the charges are far away from each other. kq q
When we bring q1 from infinity to its position, no work p = 2 1 2 = m1v1 = m2v2
r0
would be done since there are no charges in vicinity
which would exert force on the charge. Now keeping p p p 2 (kq1q2 )
q1 \ vrel = v1 + v2 = + = =
q1 fixed q2 is brought till separation m1 m2 r0
r12, so work would be required to
r12 r31 Alternatively, in COM (centre of mass) frame
be done against the force of q1. Now
keeping both q1 and q2 fixed, q3 is 1 2
r23
KEsystem = vrel
moved for which work would again q2 q3 2
be done against forces of q1 and q2. KEi + PEi = KEf + PEf
\ Usystem = W1 + W2 + W3
kq1q2 1 2
= vrel
kq q kq q kq q kq q kq q kq q r0 2
=0+ 1 2 + 1 3 + 2 3 = 1 2 + 2 3 + 3 1
r12 r13 r23 r12 r23 r31
2 (kq1q2 )
A close introspection shows us that we need to form as vrel =
r0
many two charge pairs that we can and then for each
kqi q j E2. The charges q1 and q2 y
two charge pair we write .
rij are fixed, while Q is A
q a q moved in a circular
For example consider four identical charges
path from A to B as 4a
placed at the vertices of a square. We have a a
4 edge pairs and 2 diagonal pairs. shown in the figure. 3a 4a
q a q Find the work required q2 q1 B x
kq 2 kq 2
\ Usystem = 4 +2 to be done to do so?
r 2r

62 Physics For you | AUGUST 16


Sol.: We observe that the seperation of Q from q1
z What if E is given in vector form,
does not change, so no work would be required to
be done for it, so consider it to be not existing for E = Ex i + E y j + Ez k ?
us.
rB
But for q2, ri = 5a and rf = 7a VB VA = (Ex i + E y j + Ez k ) (dxi + dy j + dzk )
1 1
rA
\ Wext = DPE = kq2Q
7a 5a xB yB zB

= Ex dx + E y dy + Ez dz
kq2Q(2) 2kq2Q x A
= = yA zA
35a 35a
Now, let us define potential difference between two where (xA, yA, zA) and (xB, yB, zB) are the (x, y, z)
points. Potential difference between two points coordinates of the respective points A and B.
A and B is defined as minimum amount of work Specifically, as a special case, if electric field is uniform,
required to be done per unit test charge in moving that is, constant in magnitude as well as direction, Ex,
the test charge from point A to point B. Ey and Ez will be constant in which case they can be
From work energy theorem (WET), taken out of integration symbol, as below.
Welectric field + Wext agent = DKE = 0 xB yB zB

Wext agent = Welectric field VB VA = Ex dx + E y dy + Ez dz
x A yA zA
rB

W W


r
Felec dr VB VA = Ex Dx + E y Dy + Ez Dz
VB VA = ext = elec . = A
q0 q0 q0 Instead of memorizing the formula, let us try to decode

rB
the expression.

q E dr

0 Ex Dx indicates component of E along x axis multiplied
VB VA =
rA with distance between the two points along x axis and
q0 this gives us ve of the potential difference between the


rB
two points. Let us find a logical way to do this.
\ VB VA = E dr Electric field along any direction multiplied with the

rA
separation between any two points in this direction
Therefore the line integral of E over any path, taken
gives the magnitude of potential difference between
with a negative sign gives us the potential difference
the two points. Whether it is positive or negative can
between the two points.
be decided by the direction of electric field.
Few note worthy points |VA VB| = (E cosq) l
z Why would anyone wish to calculate potential E
No t e t h e d i re c t i o n o f
difference between two points? E cosq from A towards B, B
A
Since if VB VA is known, multiplying it with a charge hence B has to beat a lower E cos
q gives the work required to be done in moving the potential l
charge form A to B, and hence it also gives the change \ VA VB = (E cosq)l
in potential energy.
z What do we mean by potential at a point?
z What does negative in the expression of VB VA on
The definition says that it refers to the minimum
RHS indicate?
work done by external agent per unit test charge in
Suppose we choose the points A and B in the direction
bringing the test charge from infinity to the point
of a field as below.
E under consideration.
Clearly E dr > 0
dr W U (r ) U ()
\ dV = VB VA = E dr < 0 A B \ V(r) V() = ext =
q0 q0
\ VB < VA
\ Electric field points in the direction of decreasing For example, potential due to a point charge Q at a
potential. distance r.

Physics For you | AUGUST 16 63


At P, This also means that the entire conductor under
P
kq0Q kq0Q electrostatic condition is equipotential. Since if E 0

VP V () = r = kQ inside the bulk, they will drift free electrons inside
r
q0 r the bulk and if E is not perpendicular to surface,
Q
kQ again drifting of free electrons along the surface would
\ VP = + V () happen, both of which are contradictions of electrostatic
r
condition.
Where V() indicates potential due to +Q at infinity.
This value is generally taken to be zero until and unless Some standard results
specified. Potential due to a charged ring at a point on its axis
One thing, we should remember here is that the Consider the ring to be Q
potential at any point is dependent upon the choice made up of large number R2 x 2
of zero potential but potential difference between any of elemental point charges
two points is independent of this choice. dq. Each dq element is at R
P
So if V() = 0, then same distance x
R2 + x 2
kQ , which is the potential for a point charge Q from point P.
VP =
r kdq
at a point P at a distance r. \ VP = dVP =
R2 + x 2
Unlike electric field which is a vector quantity, electric
k kQ
potential is a scalar quantity, hence the positioning VP =
R +x 2 2 dq = R2 + x 2
of the charge is immaterial as long as the separation
VP
remains unchanged.
r1 P
Potential at any point kQ
Potential at centre
q1 R
due to multiple point
charges is a scalar r2

sum of potential due x


rn
to individual point q2 Potential due to a uniformly charged thin spherical shell
charges. We know from shell theorem that the field of a
\ At point P, uniformly charged sphere matches with a point charge
qn
kq kq kq for external points so the results of potential will match
V = 1 + 2 + ....... + n too, i.e., for external points, we can assume the entire
r1 r2 rn
charge on sphere to be concentrated at the centre.

The locus of all such point in space which are at identical But for internal points E = 0 , hence potential will
potential is said to be equipotatial surface. become constant and equal to that of the surface since
There cannot be any non-zero component of electric no further work is required to be done in moving the
field along equipotential surface else it would create a test charge from surface to internal point.
potential difference which is a contradiction. + + +
+ +

Hence E will always be perpendicular to equipotential


+
+

+
+

surfaces. R kQ
, rR
+
+

Saying a surface is equipotential is different from saying R


+
+

that the entire region of a space is equipotential. In the V (r)=


+
+


+
+
+ kQ
1st case, E may or may not be present but if it is present, , rR
+ +

V r
it has to be perpendicular to the surface but in 2nd kQ
Vs
case E = 0 everywhere in the region else in whichever R
1
V
direction E 0 along this direction potential difference r
would be created which is a contradiction. r

64 Physics For you | AUGUST 16


Potential due to a uniformly charged solid sphere In general,
For external points we can again imagine the entire
charge to be concentrated at the centre.
But for internal points let us see + +
+
how do we find it +
+
+ +
+ +
+
+
R +
P is an internal point at a distance +
+
+
+
+
1+ r + +
+ +
r from the centre (r < R) we have +
+ GroundinG concePt
+ +
divided the entire sphere into two 2+
+ P
Grounding any conductor gives two informations
+
parts namely 1 and 2 . +
z The potential of the conductor becomes zero.
For 1 , the point is external point for which z Earth is considered to be an infinite source as well as
potential. sink of electrons, hence as many electrons can flow
3Q 4 r 3 in either direction as required by the conductor to
k
kq 3 kQ make its potential equal to zero.
VP1 = 1 = 4 R 3 = 3 r 2
r r R MisconcePtion
For 2 , we can imagine P to be an internal point for Grounding a conductor means its charge becomes
large number of thin concentric shells. zero. Well this is true, had the conductor been isolated
Charge dq in the elemental shell of radius x and but not if there were other charged conductors in its
vicinity. Let me explain.
Q 3Q 2
thickness dx is dq =
4
2
(
4 x dx = 3 x dx
R
) Case-I
R
3 Isolated grounded conductor (initially charged with Q) :
3 The entire charge flows to earth,
\ VP 2 = dV2 since if the conductor has any +
+ +Q
+
kdq non zero charge, it will create + qflown = Q
= 2 a field which will create a +
+
+
x
R r x dx
potential difference between
k 3Q
= 3 x 2dx infinity and conductor which is again a contradiction
r
x R since at infinity also the potential is zero.
P
3kQ 2 2 Case-II
= (R r )
2R 3 Concentric shell arrangement : +Q
kQ 3 kQ 2 2 Outer shell with charge Q and R
\ VP = VP1 + VP 2 = 3 r 2 + (R r ) q
R 2 R3 inner shell initially uncharged qflown = q
kQ and later grounded. r
VP = 3 [3R2 r 2 ] (As r < R)
2R Let us assume a charge q has
flown from ground to the
Potential at centre, for r = 0, inner shell.
3 kQ kQ kq r
VC = \ Vinnershell = 0 + = 0 q = Q .
2 R R r R
kQ This clearly shows that charge on grounded conductor
and potential at surface, VS = is non-zero.
R
3
\ VC = VS , this result is applicable only if potential MPP-2 CLASS XI Answer KEY
2
at infinity is taken as zero. V 1. (c) 2. (a) 3. (d) 4. (c) 5. (d)
But since the difference in VC 6. (b) 7. (c) 8. (d) 9. (b) 10. (c)
parabolic
potential is not dependent 11. (b) 12. (c) 13. (a) 14. (a) 15. (d)
VS
on this choice hyperbolic 16. (a) 17. (b) 18. (a) 19. (a) 20. (a,d)
kQ 21. (b,c,d) 22. (a,b,c) 23. (b,c,d) 24. (1) 25. (4)
VC VS = R r
2R 26. (5) 27. (c) 28. (a) 29. (c) 30. (b)

66 Physics For you | AUGUST 16


chapterwise McQs for practice
Useful for All National and State Level Medical/Engg. Entrance Exams
Wave oPtics
b2 b2 b2 3b2
1. In Youngs double slit experiment, the slits are 2 mm (a) l = , (b) l = ,
d 3d 2d 2d
apart and are illuminated by photons of two
wavelengths l1 =12000 and l2 = 10000 . At 2b2 3b2
(c) l = (d) l =
what minimum distance from the common central 3d 4d
bright fringe on the screen 2 m from the slit, will a 5. An unpolarised beam of light is incident on a set
bright fringe from one interference pattern coincide of four polarising plates such that each plate makes
with a bright fringe from the other? an angle of p/3 with preceding sheet. The light
(a) 3 mm (b) 8 mm (c) 6 mm (d) 4 mm transmitted through the combination is
(a) 1/128 (b) 1/256 (c) 1/64 (d) 1/32
2. Air has refractive index 1.0003. The thickness of air
column, which will have one more wavelength of 6. In a double slit pattern (l = 6000 ), the 1st order
yellow light (6000 ) than in the same thickness of and 10th order maxima fall at 12.50 mm and
vacuum is 14.75 mm from a particular reference point. If l is
(a) 2 mm (b) 2 cm (c) 2 m (d) 2 km changed to 5500 , other arrangements remaining
the same, position of 10th order maxima will be
3. The percentage of incident unpolarised light passing (a) 12.25 mm (b) 14.55 mm
through the two Nicol prisms oriented with their (c) 15.50 mm (d) 16.55 mm
principal planes making an angle q is 25. Then q is
7. The time period of rotation of the sun is 25 days
(a) 60 (b) 30 (c) 45 (d) 90
and its radius is 7 108 m. The Doppler shift for the
4. White light is used to illuminate the two slits in a light of wavelength 6000 emitted from the surface
Youngs double slit experiment. The separation of the sun will be
between slits is b and the screen is at a distance d(>> b) (a) 0.04 (b) 0.40 (c) 4.00 (d) 40.0
from the slits. At a point on the screen directly in
front of one of the slits, certain wavelengths are 8. A rocket is going towards moon with a speed v. The
missing. Some of these missing wavelengths are astronaut in the rocket sends signals of frequency u
towards moon and receives them back on reflection
from the moon. What will be the frequency of signal
received by astronaut? (Take v << c)
cu cu 2vu
(a) (b) (c) (d) 2cu
c v c 2v c v
9. A glass slab of thickness 8 cm contains the same
number of waves as 10 cm of water, when both are
traversed by the same monochromatic light. If the

Physics for you | August 16 67


refractive index of water is 4/3, the refractive index other. Then in the interference pattern
of glass is (a) the intensities of both the maxima and the
(a) 5/4 (b) 3/2 (c) 5/3 (d) 16/15 minima increase
10. Assuming the diameter of the eye pupil to be 2.0 mm. (b) the intensity of maxima increases and the
The smallest angular separation at which two point minima has zero intensity
objects can be distinctly seen when viewed in light of (c) the intensity of maxima decreases and that of
wavelength 6000 is minima increases
(a) 3.66 104 rad (b) 2.56 104 rad (d) the intensity of maxima decreases and the
(c) 1.09 104 rad (d) 4.51 103 rad minima has zero intensity.
11. Two coherent light sources A and B are at a 15. A beam of unpolarized light of intensity I0 is passed
distance 3l from each other (l = wavelength). The first through a tourmaline crystal A and then
minimum distance from A on the x-axis at which through another tourmaline crystal B and oriented
the interference is constructive is so that its principal plane is parallel to that of A. The
intensity of final emergent light is I. The value of I is
(a) I0/2 (b) I0/4
(c) I0/8 (d) none of these
Dual nature of matter anD raDiation
16. A parallel beam of light is incident normally on
(a) 3l (b) 4l (c) 5l (d) 8.75l a plane surface absorbing 40% of the light and
reflecting the rest. If the incident beam carries 60 W
12. In Fraunhoffer diffraction at a single slit of width of power, the force exerted by it on the surface is
d with incident light of wavelength 5500 , the (a) 3.2 108 N (b) 3.2 107 N
first minimum is observed at angle of 30. The first 7
(c) 5.12 10 N (d) 5.12 108 N
secondary maximum is observed at angle q, equal
17. A photon of energy E ejects a photoelectrons from
to
a metal surface whose work function is W0. If this
1 1
(a) sin 1 (b) sin 1 electron enters into a uniform magnetic field of
2 4
induction B in a direction perpendicular to the field
3 3 and describes a circular path of radius r, then the
(c) sin 1 (d) sin 1
4 2 radius r, is given by
13. Two ideal slits S1 and S2 are at 2m(W0 E ) 2e(E W0 )
a distance d apart and (a) (b)
eB mB
illuminated by light of
wavelength passing through 2m(E W0 ) 2mW0
(c) (d)
an ideal source. Slit S placed eB eB
on the line through S2 as 18. What wavelength is corresponding to a beam of
shown in figure. The distance electrons whose kinetic energy is 100 eV?
between the planes of slits (h = 6.6 1034 J sec; 1 eV = 1.6 1019 J;
and the source slit is D. A screen is held at a distance me = 9.1 1031 kg)
D from the plane of the slits. The minimum value of (a) 1.2 (b) 2.4 (c) 3.6 (d) 4.8
d for which there is darkness at O is
19. Find the maximum kinetic energy of photoelectrons
3lD lD
(a) (b) lD (c) 3lD (d) emitted from the surface of lithium of work function
2 2
2.39 eV, when exposed with light wave of electric
14. In a double slit experiment, instead of taking slits of field given by E = E0 sin [1.57 107 m1(ct x)]
equal widths, one slit is made twice as wide as the [Given h = 6.63 1034 J s]

68 Physics for you | August 16


(a) 3.11 eV (b) 2.39 eV
(c) 1.56 eV (d) 0.71 eV
20. One milli watt of light of wavelength 4560 is
incident on a cesium surface of work function 1.9 eV.
Given that quantum efficiency of photoelectric
emission is 0.5%, Plancks constant, h = 6.62 1034 J s,
velocity of light c = 3 108 m s1, the photoelectric
current liberated is
(a) 1.836 106 A (b) 1.836 107 A
(a) 1 1014 Hz (b) 1.3 1014 Hz
(c) 1.836 105 A (d) 1.836 104 A
(c) 2.3 1014 Hz (d) 2.7 1014 Hz
21. Given that a photon of light of wavelength 10000
25. A silver sphere of radius 1 cm and work function
has an energy equal to 1.243 eV. When light of
4.7 eV is suspended from an insulating thread in
wavelength 5000 and intensity I0 falls on a
free-space. It is under continuous illumination
photoelectric cell, the surface current is 0.40 106 A
of light of wavelength 200 nm. As photoelectrons
and the stopping potential is 1.36 V, then the work are emitted, the sphere gets charged and acquires a
function is potential. The maximum number of photoelectrons
(a) 0.43 eV (b) 0.55 eV emitted from the sphere is A 10Z (where 1 < A < 10).
(c) 1.12 eV (d) 1.53 eV The value of Z is
22. A cathode ray tube contains a pair of parallel metal (a) 8 (b) 10 (c) 7 (d) 2
plates 1.0 cm apart and 3.0 cm long. A narrow 26. The energy that should be added to an electron
horizontal beam of electrons with a velocity of to reduce its de-Broglie wavelength from 1 nm to
3 107 m s1 is passed down the tube mid way 0.5 nm is
between the two plates. When the potential (a) four times the initial energy
difference of 550 V is maintained across the plates, (b) equal to the initial energy
it is found that the electron beam is so deflected (c) twice the initial energy
that it just strikes the end of one of the plates. Then (d) thrice the initial energy.
the specific charge of an electron in C kg1 is 27. We wish to see inside an atom. Assuming the atom to
(a) 1.8 10 9
(b) 1.8 10 11
have a diameter of 100 pm [1 picometre (pm) = 1012 m],
(c) 3.6 1012 (d) 3.6 1014 this means that one must be able to resolve a width
of say 10 pm. If an electron microscope is used, the
23. Silver has a work function of 4.7 eV. When
minimum electron energy required is about
ultraviolet light of wavelength 100 nm is incident
(a) 1.5 keV (b) 15 keV
upon it, a potential of 7.7 volt is required to stop the
(c) 150 keV (d) 1.5 MeV
photoelectrons from reaching the collector plate.
How much potential will be required to stop the 28. In a photoemissive cell, with exciting wavelength
photoelectrons when light of wavelength 200 nm is l, the fastest electron has speed v. If the exciting
incident upon silver? 3l
wavelength is changed to , the speed of the
(Given hc = 12375 eV ) 4
(a) 0.5 V (b) 1.5 V fastest emitted electron will be
(c) 2.35 V (d) 3.85 V (a) less than v(4/3)1/2 (b) v(4/3)1/2
24. Figure represents a graph of kinetic energy of most (c) v(3/4)1/2 (d) greater than v(4/3)1/2
energetic photoelectrons Kmax (in eV) and frequency 29. A source S1 is producing 1015 photons per second of
u for a metal used as cathode in photoelectric wavelength 5000 . Another source S2 is producing
experiment. The threshold frequency of light for 1.02 1015 photons per second of wavelength 5100 .
the photoelectric emission from the metal is The, (power of S2)/(power of S1) is equal to

70 Physics for you | August 16


(a) 1.00 (b) 1.02 (c) 1.04 (d) 0.98 Using law of Malus,
intensity of polarised light transmitted from second
30. Photoelectric effect experiments are performed
prism is
using three different metal plates p, q and r having 1
work functions fp = 2.0 eV, fq = 2.5 eV and fr = 3.0 eV, I2 = I1 cos2 q = I0 cos2 q
2
respectively. A light beam containing wavelengths of I2 25 1
2
550 nm, 450 nm and 350 nm with equal intensities cos q = 2 = 2 =
I0 100 2
illuminates each of the plates. The correct I-V graph
1
for the experiment is cos q = ; q = 45
2
1/2
2 2 1/2
b2
4. (a) : As per question, S2P = (d + b ) = d 1 +
d2
(a) (b) b
2
b 2
= d 1 + =d+
2 2d
2d
Path difference = S2P S1P
b2
x = d + d = b2 / 2d
(c) (d) 2d
l b2
For missing wavelengths (2n 1) = x =
solutions 2 2d
1. (c) : Here, d = 2 mm = 2 103 m, l1 = 12000 , b2 b2
For n = 1, l = , for n = 2,l =
l2 = 10000 , D = 2 m, x = ? d 3d
Let n1th bright fringe from l1 coincide with n2th 5. (a) : Here, I0 is the intensity of unpolarised light
bright fringe from l2 falling on first plate. Then intensity of polarised
\ n1l1 = n2l2 light coming from first plate is
n1 l2 10000 5 I1 = I0/2
or = = =
n2 l1 12000 6 Using Malus law,
For minimum distance x, n1 = 5 and n2 = 6 I2 = I1 cos2 p/3 = I1/4
l D 5 (12000 1010 ) 2 I I
xmin = n1 1 = I3 = I2 cos2 p / 3 = 1 (1 / 2)2 = 1
\ 4 16
d 2 103
I I
3
= 6 10 m = 6 mm I 4 = I3 cos2 p / 3 = 1 (1 / 2)2 = 1
16 64
2. (a) : Here ma = 1.0003 and mv = 1 I0 1
l v ma I4 = = I
\ = = 1.0003 2 64 128 0
l a mv 6. (b) : Here, l = 6000
x = lv n = la (n + 1) Between 1st order and 10th order maxima, there are
n + 1 lv 9 fringes. Therefore,
= = 1.0003
n la 9 b = 14.75 12.50 = 2.25 mm
1 1 104 2.25
or 1 + = 1.0003 ; n = = b= = 0.25 mm
n 0.0003 3 9
104 When wavelength is changed to 5500 , the new
\ x = lv n = 6000 107 mm = 2 mm
3 fringe width becomes,
3. (c) : If I0 is intensity of unpolarised light falling on 5500
b = b = 0.92 0.25 mm
first prism, then intensity of polarised light from 6000
first prism, b = 0.23 mm
1 The position of central maxima or zero order
I1 = I0 maxima remains the same, which was earlier at
2
Physics for you | August 16 71
12.50 0.25 = 12.25 mm 8
\ New position of 10th order maxima would be \ Number of waves in 8 cm of glass = ,
lg
y = 12.25 + 10 b = 12.25 + 10 0.23 = 14.55 mm
10
v l and number of waves in 10 cm of water =
7. (a) : Dopplers shift, dl = l = r lw
c c
As the number of waves contained in both the
2p l medium are same (Given)
= r
T c
\ 8 10 l 10 5
7 108 2 22 6000 = or w = =
= = 0.04 l g lw lg 8 4
25 24 60 60 7 3 108 c c
8. (b) : In this situation, we can assume as if both Now m g = and mw =
vg vw
the source and observer are moving towards each m g vw ulw 5
other with a speed v \ = = =
mw v g ul g 4
\ As the no. of waves contained in both the
medium are same (Given) 5 5 4 5
m g = mw = =
4 4 3 3
(c + v )u (c v )(c + v )u
u = = 10. (a) : Here, l = 6000 = 6 107 m,
c v (c v )2 D = 2.0 mm = 2.0 103 m
(c 2 v 2 )u c2u The limit of resolution of the eye,
= = ( v 2 << c 2 )
c 2 + v 2 2cv c 2 2cv 1.22 l 1.22 6 107
dq = = = 3.66 104 rad.
cu D 2.0 103
u =
c 2v 11. (b) : Let interference be constructive at any point P
9. (c) : Let the wavelength of monochromatic light in on X-axis; AP = x
glass be lg cm, and in water be lw cm. BP = AB2 + AP 2 = (3l)2 + x 2

72 Physics for you | August 16


\ Intensities of both, maxima and minima
Path difference = BP AP = (3l)2 + x 2 x = l increase.
(for constructive interference)
15. (a) : Intensity of polarised light from tourmaline
8l 2
9l2 + x2 = (l + x)2 or 8l2 = 2lx \ x = = 4l I
2l crystal A = 0
2
12. (c) : Using the relation, d sin q = n l As principal plane of crystal B is parallel to that of A,
d sin30 d (here q = 30) therefore, intensity of final emergent light is
l= =
1 2 I = I0/2 cos2 0 = I0/2
For first secondary maximum
16. (b) : Momentum of incident light per second
l 3l 3d
d sin q = (2n + 1) = = E 60
2 2 4 p1 = = = 2 107
c 3 108
3 3
sin q = q = sin 1 Momentum of reflected light per second
4 4 60 E 60 60
13. (d) : As it is clear from figure p2 = = = 1.2 107
100 c 100 3 10 8

\ Force on the surface = Change in momentum


per second
= p2 (p1) = p2 + p1
= (1.2 + 2) 107 = 3.2 107 N
17. (c) : We know that Einstein equation,
1 2(E W0 )
E = W0 + mv 2 or v=
Path difference between the waves reaching O from S 2 m
and a charged particle placed in uniform magnetic
Dx = Dx1 + Dx2
field experience a force,
= (SS1 SS2) + (S1O S2O)
mv 2 mv 2 mv 2m(E W0 )
= D 2 + d 2 D + D 2 + d 2 D F= evB = r = =
r r eB eB
1/2 d2 p2
d2 18. (a) : We know that kinetic energy, Ek = ;

= 2 D 1 + D = 2 D 1 + 2 D 2m
D 2 2D so p = 2mEk
d2 De-Broglie wavelength,
= 2 = d2/D
2D h h 6.6 1034
l= = =
For darkness at O, p 2mEk 2 9.1 1031 100 1.6 1019
l l
Dx = (2n 1) = (2 1 1) = 1.22 1010 m = 1.2
2 2
d2 l lD 19. (d) : Here, E = E0 sin [1.57 107 m1 (ct x)]
= or d = Comparing it with the equation of harmonic wave
D 2 2
in electric field;
14. (a) : In interference, it is known that
I max = ( I1 + I2 )2 , and I min = ( I1 I2 )2
When widths of both the slits are equal, MPP-2 CLASS XII ANSWER KEY
I1 = I2 = I \ Imax = 4I and Imin = 0 1. (a) 2. (a) 3. (d) 4. (b) 5. (a)
When width of one of the slits is increased, intensity
6. (d) 7. (a) 8. (d) 9. (c) 10. (c)
due to that slit would increase, while that due to the
other will remain the same. 11. (c) 12. (b) 13. (b) 14. (a) 15. (b)
Let I1 = I and I2 = nI where n > 1 16. (b) 17. (b) 18. (a) 19. (b) 20. (a,c)
\ I max = I (1 + n )2 > 4 I 21. (a,b,d) 22. (a,b,c) 23. (a,c,d) 24. (7) 25. (6)
2
I min = I ( n 1) > 0 26. (6) 27. (b) 28. (a) 29. (b) 30. (a)

Physics for you | August 16 73


2p 23. (b) : Here, l1 = 100 nm = 1000 and
E = E0 sin (vt x ), we have
l l2 = 200 nm = 2000
2p 2p Now
= 1.57 107 or l = m
l 1.57 107 hc 12375 eV
hc E1 = = = 12.375 eV
Maximum kinetic energy (in eV) = f0 l1 1000
el 12375
and E2 = = 6.1875 eV
(6.63 1034 ) (3 108 ) 1.57 107 2000
= 2.39
1.6 1019 (2 3.141) As eVs = E1 f0
= 3.1 2.39 = 0.71 eV \ 7.7 eV = 12.375 f0 ...(i)
hc and eVs = 6.1875 f0 ...(ii)
20. (a) : Energy of photon =
Power of lamp = P l Subracting (ii) from (i), we get
P (7.7 Vs)e = 6.1875 eV
Number of photons emitted per sec, n = 7.7 Vs = 6.1875
hc / l
\ Number of photoelectrons emitted per sec or Vs = 7.7 6.1875 = 1.5 V
0. 5 1 Pl 24. (d) : From graph, u = 1015 Hz,
= n= Kmax = 3 eV = 3 1.6 1019 J
100 200 hc
1 Pl As Kmax = hu hu0 or hu0 = hu Kmax
\ Photoelectric current = e
200 hc K 3 1.6 1019
\ u0 = u max = 1015
1 (103 ) (4560 1010 ) (1.6 1019 ) h 6.63 1034
=
200 (6.62 1034 ) (3 108 ) = 2.7 1014 Hz
= 1.836 106 A 25. (a) : Using Einsteins photoelectric equation,
hc 1240
21. (c) : We know that energy of photon, eV0 = W = 4. 7
l 200
hc 1 E l
E= or E \ 2 = 1 = 6.2 4.7 = 1.5 eV
l l E1 l2
\ Stopping potential, V0 = 1.5 V
l1 10000 1 q 1 ne
or E2 = E1 = 1.243 = 2.486 eV But V0 = =
l2 5000 4 p0 r 4 p0 r
1 2
Now hv f0 = mvmax = eVs 9 109 n 1.6 1019
2 \ 1. 5 =
or f0 = E2 eVs = 2.486 1.36 = 1.126 1.12 eV 102
1 1.5 102
22. (b) : Here, y = cm = 0.5 102 m; or n= = 1.04 108
2 10
9 1.6 10
l = 3 cm = 3 102 m; v = 3 107 m s1; On comparing with A 10Z, we get Z = 8
d = 1 cm = 102 m; V = 550 V h 1
Time to cross the tube, 26. (d) : As, l = i.e., l
2mK K
l 3 102 Given li = 1 nm, lf = 0.5 nm
t= = = 109 s
v 3 107
li Kf
1 1 Ee \ =
y = at 2 = t 2 lf Ki
2 2 m
e 2y 2y 2 yd 1 109 Kf Kf
or =
2
=
2
= or 9
= or = (2)2 = 4
m Et (V / d )t Vt 2 0.5 10 Ki Ki
2 2 or Kf = 4Ki
2 0.5 10 10
= = 1.8 1011 C kg 1
550 (10 ) 9 2 \ Energy to be added = Kf Ki = 4 Ki Ki = 3Ki

74 Physics for you | August 16


h h2 29. (a) : Energy emitted per sec by S1,
27. (b) : l = or l2 = hc
2mK 2mK P1 = n1
l1
h 2
(6.6 1034 )2 Energy emitted per sec by S2,
K= = J
2ml2 2 9.1 1031 (10 1012 )2 P2 n2 l1 1.02 1015 5000
6.6 6.6 1015 \ = = = 1. 0
= k eV 15 k eV P1 n1 l2 1015 5100
2 9.1 1.6 1016 30. (a) : Energies of incident photons of different
28. (d) : According to Einsteins photoelectric equation, wavelengths will be
when the exciting wavelength is l, hc 1240
E(550 nm) = = = 2.25 eV
hc 1 l 550
hu = = hu0 + mv 2 1240
l 2 E(450 nm) = = 2.75 eV
3l 450
When the exciting wavelength is ,
4 1240
hc 1 E(350 nm) = = 3.54 eV
= hu0 + mv 2 350
3l / 4 2
In case of plate p, all radiations will cause
1 4 hc 4 1
or mv 2 = hu0 = (hu0 + mv 2 ) hu0 photoelectric emission.
2 3 l 3 2 In case of plate q, wavelengths 450 nm and 350 nm
1 1 2 will cause photoelectric emission.
or mv 2 = hu0 + mv 2
2 3 3 In case of plate r, only wavelength 350 nm will cause
1 2 2 2 4 photoelectric emission.
mv > mv or v 2 > v 2 Hence saturation current will be maximum for plate
2 3 3
or v > v(4/3)1/2 p, intermediate for q and minimum for r.

IIT dreams: 823 SC students in JEE (Advanced)


general list, 22 in top 2,000
This years Joint Entrance Exam (Advanced) exams for entry into the IITs saw 823 SC students make it to the Common Rank List in which
candidates across categories are given ranks. In other words, these students got a general rank without using the 50 per cent relaxation
in marks they were entitled to.
According to data provided to The Indian Express by IIT Guwahati, the organising IIT for this years JEE, 22 of these 823 students ranked
between 1 and 2,000 in the Common Rank List. To be included in the CRL, students irrespective of category need to score a minimum
10 per cent in each subject and an aggregate 20 per cent.
The 823 qualifiers are a marked increase from the 515 SC students who made the CRL last year. They account for 11.21 per cent of the
total 7,339 SC students who had cleared JEE (Advanced), while the proportion of SC students who had got general ranks last year had
been 20 per cent of the total 2,570 who had cleared the exam.
Until last year, SC students needed to score at least 3.5 per cent in each subject and an aggregate of 12.25 per cent to qualify for IITs.
This year, the minimum aggregate percentage that SC students need to qualify has been brought down to 10, said 2015 JEE chairperson
Professor P V Balaji of IIT Bombay. This explains why over 7,000 SC students qualified this year compared to 2,570 in 2015.
The IITs have 10,500 seats. Of these, 7.5 per cent are reserved for STs, 15 per cent for SCs, 27 per cent for OBCs and the remaining
50.5 per cent for general category students.
While these 823 students are eligible for the open seats [general category seats], it is likely that a majority of them will take admission
under the SC category. This is because they have better chances of getting their preferred institute and course in the reserved seats.
Unless someone from among these 823 has a rank in the top 20 [CRL], these students are unlikely to get into the college of their choice,
said Prof K V Krishna, chairman, JEE 2016, of IIT Guwahati.
Also, not all the 7,000-plus SC students who have qualified will make it to the IITs remember, there are only 10,500 seats in all. But
they can get admission to the other engineering colleges in the country through their JEE (Main) ranks, Krishna said.
Among ST students, 9,537 students appeared this year and 2,392 of them (25 per cent) qualified. Last year, 1,745 of 7,736 ST students
(22.5 per cent) had cleared the exam. The cutoff for STs was the same as for SCs in both 2015 and 2016.
This year, 212 ST students have found a place in the common merit list as opposed to 108 last year. Of the 212, two had ranks between
1 and 2,000 in the CRL
The JEE (Advanced) 2016 results were declared on June 12. Courtesy : The Indian Express

Physics for you | August 16 75


Class XII

T his specially designed column enables students to self analyse their


extent of understanding of specified chapters. Give yourself four
marks for correct answer and deduct one mark for wrong answer.
Self check table given at the end will help you to check your
readiness.

Electrostatic Potential and Capacitance


Total Marks : 120 Time Taken : 60 min
NEET / AIIMS / PMTs molecule at a point 52.0 nm away along the axis of
Only One Option Correct Type the dipole? Assume V = 0 at infinity.
1. Two identical positive charges are placed on the (a) 10.3 mV (b) 16.3 mV
y-axis at y = a and y = + a. The variation of (c) 20.3 mV (d) 26.3 mV
electric potential V along x-axis is shown by graph 5. Figure shows the variation of electric field intensity
E versus distance x. What is the potential difference
(a) (b) between the points at x = 2 m and at x = 6 m from O ?

(c) (d)
(a) 30 V (b) 60 V (c) 40 V (d) 80 V
2. In a typical lightning flash, the potential difference 6. A capacitor of 4 mF is connected as shown in the
between discharge points is about 1.0 109 V and circuit. The internal resistance of the battery is
the quantity of charge transferred is about 30 C. If 0.5 W. The amount of charge on the capacitor plates
all the energy released could be used to accelerate will be
a 1200 kg automobile from rest, what would be the
final speed of the automobile?
(a) 7100 m s1 (b) 3600 m s1
(c) 9500 m s 1 (d) 6800 m s1
3. Two capacitors each having capacitance C and
breakdown voltage V are joined in series. The (a) 0 (b) 4 mC (c) 16 mC (d) 8 mC
capacitance and the breakdown voltage of the 7. Three charges q, 2q and 8q are to be placed on a
combination will be 9 cm long straight line. Where the charges should
(a) 2C and 2V (b) C/2 and V/2 be placed so that the potential energy of this system
(c) 2C and V/2 (d) C/2 and 2V is minimum.
4. The ammonia molecule NH3 has a permanent (a) charge q between charges 2q and 8q and 3 cm
electric dipole moment equal to 1.47 D, where from charge 2q.
D is the debye unit with a value of 3.34 1030 C m. What (b) charge q between charges 2q and 8q and 5 cm
will be the electric potential due to an ammonia from the charge 2q.

76 Physics For you | AUGUST 16


(c) charge 2q between charges q and 8q and 5 cm Assertion & Reason Type
from the charge q. Directions : In the following questions, a statement of
(d) charge 2q between charges q and 8q and 7 cm assertion (A) is followed by a statement of reason (R). Mark
from the charge q. the correct choice as :
8. Which of the following statements is incorrect (a) If both assertion and reason are true and reason is the
regarding equipotential surfaces? correct explanation of assertion.
(a) Equipotential surfaces are closer in regions (b) If both assertion and reason are true but reason is not
of large electric fields compared to regions of the correct explanation of assertion.
lower electric fields. (c) If assertion is true but reason is false.
(b) Equipotential surfaces will be more crowded (d) If both assertion and reason are false.
near sharp edges of a conductor. 13. Assertion : When two conductors charged to
(c) Equipotential surfaces will be more crowded different potentials are connected to each other, the
near regions of large charge densities. negative charge always flows from lower potential
(d) Equipotential surfaces will always be equally to higher potential.
spaced. Reason : In the process of charging there is always a
9. An electric dipole of length 2 cm is placed with its flow of electrons only.
axis making an angle of 30 to a uniform electric 14. Assertion : The potential difference between two
field 105 N C1. If it experiences a torque of concentric spherical shells depends only on the
10 3 N m, then the potential energy of the dipole is charge of inner shell.
(a) 10 J (b) 20 J (c) 30 J (d) 40 J Reason : The electric field in the region in between
two shells depends on the charge of inner shell and
10. The arrangement consists
electric field is the negative of potential gradient.
of four identical plates.
P 15. Assertion : If the distance between parallel pates
Each plate has the area A
of a capacitor is halved and dielectric constant
and the plate separation is
is made three times, then the capacitance becomes
d. The equivalent capacitance Q
six times.
between P and Q is
Reason : Capacitance of the capacitor does not
e A 3e0 A 2 e0 A 3 e0 A
(a) 0 (b) (c) (d) depend upon the nature of the material of the
2d d 3 d 2 d plates.
11. Three charges q, +q and +q are situated in x-y plane
JEE MAIN / JEE AdvANcEd / PETs
at points (0, a) (0, 0) and (0, a) respectively. The
Only One Option Correct Type
potential at a point distant r (r > a) in a direction
making an angle q from y-axis will be 16. A capacitor is made of two circular plates of
kq radius R each, separated by a distance d<<R.
kq
(a) (b) 2 (2a cos q) The capacitor is connected to a constant
r r voltage. A thin conducting disc of radius r << R
kq 2a cos q kq and thickness t << r is placed at a centre of the bottom
(c) 1 + (d)
r r a plate. What is the minimum voltage required to lift
12. A charge Q has been divided on two concentric the disc if the mass of the disc is m?
conducting spheres of radii R1 and R2 (R1 > R2) mgd mgd 2
such that the surface charge densities on both the (a) (b)
e0r e0r 2
spheres are equal. The potential at their common
centre is mgr 2 mgr
(c) (d)
1 Q(R12 + R22 ) 1 Q(R1 + R2 )
e0d 2 e0d
(a) (b)
4 e0 (R1 + R2 ) 4 e0 (R12 + R22 )
17. Two conducting spheres, one of radius 5.88 cm and
1 QR1R2 1 Q(R1 + R2 ) the other of radius 12.2 cm, each have a charge of
(b) (d)
4 e0 (R1 + R2 ) 4 e0 R1R2 28.6 nC and are very far apart. If the spheres are

Physics For you | AUGUST 16 77


subsequently connected by a conducting wire, then Given that K = 1 q which of the following
,
the potential of each sphere, will be 4 e0 L2
(a) 2350 V (b) 2850 V statements are correct?
(c) 1530 V (d) 1810 V
18. A total amount of positive charge Q is spread onto
a non-conducting, flat, circular annulus of inner
radius a and outer radius b. The charge is distributed
so that the charge density (charge per unit area) is
given by s = k/r3, where r is the distance from the
centre of the annulus to any point on it and k is a
constant. The potential at the centre of the annulus
will be (a) The electric field at O is 6K along OD.
Q a +b Q a +b (b) The potential at O is zero.
(a) (b) (c) The potential at all points on the line PR is
8e0 ab 4 e0 ab
same.
Q b Q a (d) The potential at all points on the line ST is
(c) ln (d) ln
4 e0a a 4 e0b b same.
19. Two equal charges are fixed at x = a and x = +a on 23. A parallel plate capacitor of plate area A and
the x-axis. Another point charge Q is placed at the separation d is charged to a potential difference
origin. The change in the electrical potential energy V and then the battery is disconnected. A slab of
of Q when it is displaced by a small distance x along dielectric constant K is then inserted between the
the x-axis is approximately proportional to plates of the capacitor so as to fill the space between
1 the plates. If Q, E and W denote respectively, the
(a) x (b) x2 (b) x3 (d) magnitude of charge on each plate, the electric field
x
between the plates (after the slab is inserted), and
More than One Options Correct Type the work done on the system, in question, in the
20. A parallel plate capacitor is formed by two identical process of inserting the slab, then
plates, each of area A, separated by a small distance e AV e0 KAV
(a) Q = 0 (b) Q =
d. One plate has a total charge Q and the other Q. d d
Neglect the edge effects then V e0 AV 2 1
(c) E = (d) W = 1
(a) The charge per unit area on the outside of each Kd 2d K
plate is (Q + Q)/2A. Integer Answer Type
(b) The charge per unit area on the outside of each
plate is (Q Q)/2A. 24. A soap bubble 10 cm in radius with a wall thickness
(c) Electric field between the plates is (Q Q)/ 2e0A. of 3.3 106 cm is charged to a potential of 70 V.
(d) Electric field between the plates is (Q + Q)/ 2e0A. The bubble bursts and falls as a spherical drop. The
potential of the drop (in kV) is
21. An electric dipole of dipole moment
^ ^ 25. When a series combination of two uncharged
p = (2 i + 3 j) mC m is placed in a uniform electric
^ ^ capacitors is connected to a 12 V battery, 173 mJ
field E = (3 i + 2 k) 105 N C1. of energy is drawn from the battery. If one of the
(a) The torque on the dipole is capacitors has a capacitance of 4 mF, the capacitance
^ ^ ^
(0.6 i 0.4 j 0.9 k) N m. of the other capacitor (in mF) is
(b) The potential energy of the dipole is 0.6 J. 26. A point charge of 2 mC moves from point P to
(c) The potential energy of the dipole is 0.6 J. point S along the path PQRS in a uniform electric
(d) If the dipole is rotated in the electric field, the field of 3 106 N C1, as shown in the figure.
maximum potential energy of the dipole is 1.3 J. The co-ordinates of points P, Q, R and S are (0, 0, 0),
22. Six point charges are kept at the vertices of a regular (1, 1, 0), (2, 0, 0) and (1, 1, 0) respectively. The work
hexagon of side L and centre O as shown in figure done (in J) by the electric field in the process is

78 Physics For you | AUGUST 16


y A B C D
(a) P R S Q, T
(b) Q P R, T S
S E
R (c) T Q S P, R
P x
(d) S Q, T R P
Q
30. Figure shows a situation in
Comprehension Type which two short dipoles of
^ ^
A solid conducting sphere of dipole moment p i and 3 p j
radius a is surrounded by a thin are placed at origin. A circle of
uncharged concentric conducting radius R with centre at origin
shell of radius 2a. A point charge is drawn as shown in figure.
q is placed at a distance 4a from Match the statements in Column I with the
common centre of conducting sphere and shell. The corresponding coordinates in Column II.
inner sphere is then grounded. Column I Column II
27. The charge on solid sphere is (A) T h e c o o r d i n a t e s (P) R 3R
of point on circle 2, 2
q q q q
(a) (b) (c) 8 (d) where potential is
2 4 16
maximum.
28. The potential of outer shell is (B) The coordinates of (Q) R 3R
q q point on circle where 2 , 2
(a) (b)
32e0a 16e0a potential is zero.
q q (C) The coordinates of (R) 3R R
(c) (d) point on circle where 2 , 2
8e0a 4 e0a
magnitude of electric
Matrix Match Type field intensity is
29. Column I gives the dependence of electric potential 1 4p
.
(V) on distance (r) due to certain charged objects 4 e0 R3
and Column II lists these objects. Match the entries (D) The coordinates of (S) 3R R
of Column I with the entries of Column II. point on circle where 2 , 2

Column I Column II magnitude of electric
(A) r 2 (P) Electric monopole field intensity is
(B) r1 (Q) Electric dipole 1 2p
(on the axial line) .
4 e0 R3
(C) r0 (R) Electric dipole A B C D
(on the equatorial line) (a) P R, S P, Q R, S
(D) r3 (S) Electric quadruple (b) Q, S Q P, S Q, R
(on the axial line) (c) P, R Q, R S P, Q
(T) Charged spherical conductor (d) P, S R Q, R P, Q
(for an inside point)
Keys are published in this issue. Search now! J

Check your score! If your score is


> 90% ExcEllEnt work ! You are well prepared to take the challenge of final exam.

No. of questions attempted 90-75% Good work ! You can score good in the final exam.
No. of questions correct 74-60% satisFactory ! You need to score more next time
Marks scored in percentage < 60% not satisFactory! Revise thoroughly and strengthen your concepts.

Physics For you | AUGUST 16 79


PHYSICS MUSING
P hysics Musing was started in August 2013 issue of Physics For You with the suggestion of Shri Mahabir Singh. The aim of Physics Musing is to augment
the chances of bright students preparing for JEE (Main and Advanced) / AIIMS / Other PMTs with additional study material.
In every issue of Physics For You, 10 challenging problems are proposed in various topics of JEE (Main and Advanced) / various PMTs. The detailed
solutions of these problems will be published in next issue of Physics For You.
The readers who have solved five or more problems may send their detailed solutions with their names and complete address. The names of those who
send atleast five correct solutions will be published in the next issue.
We hope that our readers will enrich their problem solving skills through Physics Musing and stand in better stead while facing the competitive exams.

single option correct type then the distance travelled by the ball before second
1. Two blocks m1 and m2 are connected with a collision at the centre
compressed spring and placed on a smooth 6 7 9 3
(a) R (b) R (c) R (d) R
horizontal surface as shown in figure. 5 5 5 2
4. Two spherical balls of radius r1 and
r2 (<r1) and of density s are tied
up with a long string and released
Force constant of spring is k. Under the influence in a viscous liquid column of lesser
of forces F1 and F2. At an instant blocks move with density r with the string just taut
common acceleration a0. At that instant force F2 is as shown. Find the tension in the string when
suddenly withdrawn. Mark correct option. terminal velocity is attained
F 3 r r1
4 4
(a) Instantaneous acceleration of m1 is a1 = a0 1 . (a) 2 (s r) g
m1 4 r2 r1
(b) Instantaneous acceleration of m2 is
2
F (b) (r24 r14 ) (s r) g
a2 = a0 + 2 . 3
m2
4
(c) Instantaneous acceleration of m1 is a1 = 0. (c) (r24 r13 ) (s r) g
3
(d) Instantaneous acceleration of m2 is a2 = 0.
4 r24 r14
2. Consider a one-dimensional collision where a body (d) (s r) g
3 r2 + r1
of mass m1 originally moving in the positive x
direction with speed v0 collides with a second body 5. For an ideal gas the equation of a process for which
of mass m2 originally at rest. The collision could be the heat capacity of the gas varies with temperature
completely inelastic, with the two bodies sticking as C = a/T ( a is a constant) is given by
together, completely elastic, or somewhere in (a) V lnT = constant
between. After the collision, m1 moves with velocity (b) VT1/(g1)ea/RT = constant

v1 , while m2 moves with velocity v2 . If m1> m2, 1

then (c) V g 1T a / RT = constant


(a) v0 < v1 < v2 (b) v2 < v1 < v0 (d) V g 1 T = constant
v0 v 6. A spherical soap bubble (surface tension = S)
(c) < v2 < v0 (d) 0 < v2 < 2v0 encloses n moles of monoatomic ideal gas. The
2 2
gas is heated slowly so that the surface area of the
3. A smooth tunnel is dug along the radius of the earth 3nR
that ends at the centre. A ball is released from the bubble increases by per unit increment in
2S
surface of earth along the tunnel. If the coefficient
of restitution is 0.2 between the surface and ball temperature. The specific heat for this process is
By Akhil Tewari, Author Foundation of Physics for JEE Main & Advanced, Professor, IITians PACE, Mumbai.

80 Physics For you | August' 16


3R 5R 7R 9R Which of the following statement(s) is/are correct?
(a) (b) (c) (d)
2 2 2 2 3 mv 2
(a) The value of elcectric field, E = .
More than one oPtion correct tyPe 4 qa
(b) Rate of work done by the electric field at P is
7. A circular wire, 10 cm in
diameter, with a slider wire 3 mv 3
.
on it, is in a horizontal 4 a
plane. A liquid film is
(c) Rate of work done by the electric field at P is
formed, bounded by the
zero.
wires, on the left side of
(d) Rate of work done by both the fields at Q is
the slider as shown in
zero.
the figure. The surface

tension of the liquid is
0.1 N m1. An applied force SOLUTION OF JULY 2016 CROSSWORD
1 2
16 mN, perpendicular to the slider, maintains the F A U L T B L O O M I N 3G 4
C
5 6 7
film in equilibrium. Ignore the sag in the film. What P
8
B
9
E G R
R C E P E T A O A E
can be the distance between point P and slider? 10
E O V I D L M E
(a) 8 cm 11 12
C O N T A C T O R P E A E P
(b) 2 cm I T C I S X T
S
(c) 5 cm S O R H
13
G R A V I T Y C E L L
(d) Slider cannot be in equilbrium. I N O 14
D A C R
15
O A N A N
8. A simple harmonic oscillator consists of a mass 16
O
N N M I O N I S A T I O N
sliding on a frictionless surface, attached to an ideal 17
C Q P G D
spring. Choose the correct statements. 18
B 19
E Q U I L I B R A N T 20
B O
(a) Quadrupling the mass will double the period. A A N U I G
21
(b) Doubling the amplitude will change the R
22
B S G G
24
S R
23
frequency. O C U L A R P N E U M A T I C S A
S S R R I O P
(c) Doubling the amplitude will double the total T B S E L P H
energy of the system. A A O 25
B E L
(d) Doubling the amplitude will quadruple the T R L
26
U M I N O U S E F F I C A C Y
total energy of the system.
9. A ray of light travelling in a transparent medium Winner (July 2016)
falls on a surface separating the medium from air at Swastik Biswas, Kolkata
an angle of incidence 45. The ray undergoes total
internal reflection. If m is the refractive index of the Solution Senders (June 2016)
medium with respect to air, select the possible value(s) Devjit Acharjee, Kolkata
of m from the following It is very interesting to solve the crossword of Physics For You.
(a) 1.3 (b) 1.4 (c) 1.5 (d) 1.6 It enhances knowledge in Physics, and being a science
lover I enjoy it very much. So, thanks for the thought of the
10. A particle of charge +q
crossword.
and mass m moving
under the influence S. Sharanya Bharghavi, Mumbai
of a uniform electric
Solution Senders of Physics Musing
field Ei and uniform
Set-36
magnetic field Bk 1. Virat Vyas, Bhopal (M.P.)
follows a trajectory 2. Suman Mondal, Kolkata (W.B.)
from P to Q as shown
in figure. The velocities at P and Q are vi and 2v j. 3. Rupesh Singh, Gorakhpur (U.P.)

Physics For you | August '16 81


Y U ASK other values of areas were adjusted so that
was kept intact.
1
4 pe0
Any good formula should lead to other discoveries

WE ANSWER
Do you have a question that you just cant get
and fit into the mosaic patterns in various fields.

4 pe0 0 = e00 = a constant whose value was
4p
1
the same as , c being the velocity of light.
answered? c2
Use the vast expertise of our mtg team to get to the This gave the clue to Maxwell to identify light as
bottom of the question. From the serious to the silly, an electromagnetic wave. The connection between
the controversial to the trivial, the team will tackle the light and electromagnetism was seen by him.
questions, easy and tough. This was a great step in the theory of light.
The best questions and their solutions will be printed in Many gems of truth are scattered everywhere. The
this column each month. great scientists recognize the truth behind the
scattered formulas.
Q1. What is the origin of 4pe0 in the formula of
0 Q2. Entropy of the universe is constantly increasing
electrostatic force and for magnetism? Can this be applied at atomic levels and other
4p
Ms. Aarohi Pandey, Mandsaur (M.P) topics in modern physics?
Ans. The original formula given by Coulomb, for the Ms. Poonam Madhe, Nasik (Maharashtra)
force between two point charges is Ans. According to White O.M. (Heat), entropy is a
kq q mathematical concept. The change in entropy is
F = 1 2 , +ve for repulsion and ve for attraction. Q
r2 where Q is the quantity of heat taken in by the
The force is proportional to the product of the T
body at temperature T.
charges and inversely proportional to the square
From Carnot cycles discussion, for a particular
of the distance between them.
change Q1 to Q2 at temperature T1 to T2, the
Later, when Gauss gave his theorem, Q Q
q1+ + change in entropy = 1 2 in each cycle. As
dS = e0 where q1 is the charge
+ q1
E T1 T2
Q1 Q2
enclosed. > , in each cycle the change in entropy is
T1 T2
But dS = 4 pr 2 for a sphere, the gaussian surface positive.
for point charges as well as spheres. Entropy increases in all irreversible changes.
q+ 1 Perfectly reversible cycle is only theoretical. There
E , the field = 1 are always leakages of heat and the insulators are
e 4 pr 2
0 never perfect. Therefore every time a machine
q1+ 1
2
This 4pr is kept as to suit the new S.I works, the sink gets hot. In normal engines,
4 pe0 r 2 entropy increases.
units. The second law of thermodynamics as stated by
Clausius is that the entropy of the world tends to
For magnetic field also one gets 0 . After all, like
4p a maximum. We should not mix modern physics
1
the constant k, and 0 are just constants. with classical physics, although later you may find
4 pe0 4p their applications.
Here it makes no difference. Q3. Why do doors and windows find it difficult to
1
But k was put and the similar constant for get closed in cold weather?
4 pe0 Mr. Arun S
0 Ans. In cold weather due to increase in humidity in the
magnetism as to suit the S.I. units. Once the
4p atomosphere water does not evaporate rapidly.
units are settled, the k was changed. So, the cellulose in the wood expands due to the
But this was a great choice, suggested by Gauss added water content and swells hence it does not
theorem, for spherical charges and later, all the fit the door frame.

82 Physics For you | august 16


hawking to announce planS to map univerSe
Stephen Hawking, one of the worlds best-known physicists, is
set to announce plans to map the entire known universe using a
supercomputing centre he founded at Cambridge University.
The Cosmos computer will plot the position and movement of billions
of galaxies, black holes, supernovas and other cosmic structures, The
Sunday Times reported.
Hawking, the author of A Brief History of Time, will detail the
plans of this super map during a talk to some of the worlds most
eminent scientists at the Starmus science conference, which begins
on June 27, 2016 in Tenerife, Spain.
Paul Shellard, Cambridge Universitys professor of cosmology and
director of the Cosmos computer centre, said it would use images of
radiation from the Big Bang captured by the European Space Agencys
Planck satellite to draw up a map of the early universe.
Planck gives us an amazing picture of the early distribution of matter
and how that led to the structure of the modern universe, he said.
The images will be augmented by data from the Dark Energy Survey,
which has a 13-feet diametre telescope in Chile, to map hundreds of
millions of galaxies and reveal the nature of the dark energy that is
accelerating the expansion of the universe.

SmalleSt microwave detector created


Researchers have created the worlds smallest and most sensitive microwave detector,
beating the previous record by fourteen fold, an advance which may help in making
ultra-sensitive cameras and accessories for emerging quantum computers.
The detector, which is smaller than a human blood cell, has a simple design of
superconducting aluminum and a single golden nanowire. For us size matters. The
smaller the better. With smaller detectors, we get more signal and cheaper price in mass
production, said Mikko Mottonen from Aalto University in Finland.
This design guarantees both efficient absorption of incoming photons and very
sensitive readout. The whole detector is smaller than a single human blood cell, the
researchers said. The new detector works at a hundredth of a degree above absolute
zero temperature.

volcanoeS go quiet before they erupt


Periods of seismic quiet occur immediately before volcanic eruptions and can thus be used
to forecast an impending eruption for restless volcanoes, scientists say.
When dormant volcanoes are about to erupt, they show some predictive characteristics -
seismic activity beneath the volcano starts to increase, gas escapes through the vent, or the
surrounding ground starts to deform.
However, until now, there has not been a way to forecast eruptions of more restless
volcanoes because of the constant seismic activity and gas and steam emissions. Diana
Roman, a volcanologist at Carnegie Institution for Science (CIS) in the US, working with a
team of scientists from Pennsylvania State University, Oxford University, the University of
Iceland found that periods of seismic quiet occur immediately before eruptions and can thus
be used to forecast an impending eruption for restless volcanoes.
The duration of the silence can indicate the level of energy that will be released when
eruption occurs.
Longer quiet periods mean a bigger bang. The team monitored a sequence of eruptions at
the Telica Volcano in Nicaragua in 2011. It is a so-called stratovolcano, with a classic-looking
cone built up by many layers of lava and ash.
They started monitoring Telica in 2009 with various instruments and by 2011 they had a
comprehensive network within four kilometres of the volcanos summit.
Courtesy : The Times of India
5
The fraction of heat lost in 500 W is of total heat
generated. 8
5 1 1
solution set-36 So, required heat generated = = J
8 10 16
1. (b) : Charge on inner shell,
qr = s(4pr2) 8. On first touch of electrometer with metal ball
Charge on outer shell, qR = s(4pR2) Vmetal = Velectrometer
Potential of given system of shells q1C1 = (Q0 q1)C2
for different values of x, Q0
1 qr qR s Net charge transfer, q1 =
(i) 0 < x r, V1 = + = (R r ) C1
4 p0 r R 0 1 + C
2
1 qr qR s r2 Similarly after second touch,
(ii) r < x R, V2 = + = ( R )
4 p0 x R 0 x q1 Q0 Q0
q2 = = and q10 = 10
C1 C 2
C1
1 qr qR s 1 +
C 1 + 1 1 +
(iii) x > R, V3 = + = (R 2 r 2 ) 2 C2 C
4 p0 x x 0 x 2

C2Q0 3 kV
1 Ne Also, C2 q10 = 1.5 kV=
2. (a) : V = 10 10
4 p0 R C1 C1
1 + C 1 + C
9 109 (N ) 1.6 1019 2 2
360 = \ N = 250000 C1
106 1/10
...(i)
1 + C = (2)
3. (a) : According to question, frequency of torsional 2
oscillation = frequency of vertical oscillation, Suppose touching process is required to be done
1 c 1 k 2c n more times to get electrometer voltage equal to
k 2c
= ; = \ R= 1 kV.
2 p I 2 p m mR 2 m k 1
C
4. (d) : BP is only because of single current. So, 1 + 1 = (3)10 +n ...(ii)
BQ is because of two currents in same direction. C2
BR is because of two currents in opposite direction. From eqns. (i) and (ii),
5. (d) : By symmetry, magnetic field cannot exist. 1 1
1 1
(2)10 = (3)10 +n ln 2 = ln 3
d2q 10 10 + n
6. t = mgR sin q = I
dt 2 10(ln 3 ln 2)
n= = 5. 8 6
For small amplitude, sinq q ln 2
Hence, electrometer reads less than 1 kV for n = 6.
d2q mgR q
\ 2
= 9. Using Newton's second law of motion,
dt MR2
f = ma
MR 4 p2 MR Taking torque about centre of mass, t 0 = Ia
\ T = 2p m=
mg T2g mg a F
FR fR = mR2
4 g 600 10 3
30 10 2 R m F
= F a
2 f = =2N
(12) g 2 f
= 5 103 kg = 5 g 10. For measurement from west end, 13(2x) + R = 200
7. Charge on capacitor, Q = 5 106 200 = 103 C For measurement from east end, 13(2 (10 x)) + R = 100
Q2 106 \ 260 + 2R = 300
Total heat generated = = 6
= 101 J R = 20 W
2C 2 5 10

84 Physics For you | AUGUST 16


Readers can send their responses at editor@mtg.in or post us with complete address by 25th of every month to win exciting prizes.
Winners' name with their valuable feedback will be published in next issue.

ACROSS
2. A cathode from which electrons are emitted
as a result of the photoelectric effect. [12]
6. The stationary electromagnetic structure of
an electric motor or generator. [6]
7. A group of magnesium alloys used to enease
uranium fuel elements. [6]
8. Commonly used to improve the frequency
response of a loudspeaker. [6]
11. The reciprocal of dynamic viscosity. [8]
14. A dimensionless number used in problems
involving both heat and mass transfer. [5, 6]
17. Any of a class of heat-resistant materials
made of ceramic and sintered metal. [6]
21. A system of axes that forms a frame of
reference. [9]
23. The derived SI unit of activity equal to one
disintegration per second. [9]
24. A self-evident proposition not requiring
demonstration. [5]
26. A type of astronomical object with a mass
between that of a large planet and a small star. [5, 5] 9. A unit of frequency. [7]
27. A transuranic radioactive chemical element with symbol 10. An abnormal transient electrical disturbance in a
Cm. [6] conductor. [5]
28. The branch of physics deals with deformation and flow 12. An apparatus for studying thermal expansion. [11]
of matter. [8] 13. A formula for calculating moments of inertia. [4, 4]
29. A negatively charged elementary particle classed as a 15. A type of switch that operates using superconductivity.
lapton. [5] [8]
30. An instrument for measuring the changes in magnetic 16. An alloy of lead and tin. [6]
flux. [9]
18. The property of allowing the passage of lines of magnetic
DOWN flux. [9]
1. An elementary particle, charged or neutral, classified as a 19. The process by which a solid undergoes plastic
hyperon. [5, 8] deformation. [5]
2. It is equal to one joule second. [6] 20. Denotes a nuclear process that absorbs energy. [9]
3. A high-pitched audio frequency tone heard in 22. A powdery metallic oxide formed when an ore has been
receivers. [4] heated. [4]
4. A temporary connection between electronic circuits. [6] 25. A unit for measuring signal speed in a computer. [4]
5. A pair of closely spaced lines in a spectrum. [7]

Physics For you | AUGUST 16 85


86 Physics For you | AUGUST 16

Potrebbero piacerti anche